Exam 2 (8-14)

Pataasin ang iyong marka sa homework at exams ngayon gamit ang Quizwiz!

Stage IV

Any distant metastasis

Stage 0

carcinoma in situ

Which signs and symptoms should a woman immediately report to her health care provider? (Select all that apply.) a. Vaginal bleeding b. Rupture of membranes c. Heartburn accompanied by severe headache d. Decreased libido e. Urinary frequency

ANS: A, B, C Vaginal bleeding, rupture of membranes, and severe headaches are signs of potential complications in pregnancy. Clients should be advised to report these signs to their health care provider. Decreased libido and urinary frequency are common discomforts of pregnancy that do not require immediate health care interventions.

Many factors, male and female, contribute to normal fertility. Which factors are possible causes for female infertility? (Select all that apply.) a. Congenital or developmental b. Hormonal or ovulatory c. Tubal or peritoneal d. Uterine e. Emotional or psychologic

ANS: A, B, C, D Female infertility can be attributed to alterations in any one of these systems along with possible vaginal-cervical factors. Although the diagnosis and treatment of infertility require considerable emotional investment and may cause psychologic stress, these are not considered factors associated with infertility. Feelings connected with infertility are many and complex. Resolve is an organization that provides support, advocacy, and education for both clients and health care providers.

Pregnancy is a hypercoagulable state in which women are at a fivefold to sixfold increased risk for thromboembolic disease. The tendency for blood to clot is greater, attributable to an increase in various clotting factors. Which of these come into play during pregnancy? (Select all that apply.) a. Factor VII b. Factor VIII c. Factor IX d. Factor XIII e. Fibrinogen

ANS: A, B, C, E Factors VII, VIII, IX, X, and fibrinogen increase in pregnancy. Factors that inhibit coagulation decrease. Fibrinolytic activity (dissolving of a clot) is depressed during pregnancy and the early postpartum period to protect the women from postpartum hemorrhage.

The nurse, responsible for providing health teaching regarding the side effects of combined oral contraceptives (COCs), should discuss what issues? (Select all that apply.) a. Gallbladder disease b. Myocardial infarction c. Hypotension d. Breast tenderness e. Dry skin and scalp

ANS: A, B, D Serious side effects include stroke, myocardial infarction, hypertension, gallbladder disease, and liver tumors. More common side effects include nausea, breast tenderness, fluid retention, increased appetite, and oily skin and scalp

Cellulitis with or without abscess formation is a common condition. The nurse is providing education for a client whose presentation to the emergency department includes an infection of the breast. Which information should the nurse share with this client? (Select all that apply.) a. Nipple piercing may be the cause of a recent infection. b. Treatment for cellulitis will include antibiotics. c. Streptococcus aureus is the most common pathogen. d. Obesity, smoking, and diabetes are risk factors. e. Breast is pale in color and cool to the touch.

ANS: A, B, D The at-risk population for breast infection shares characteristics such as large breasts, obesity, previous surgeries, sebaceous cysts, smoking, diabetes, and recent nipple piercing. The most common pathogen is Staphylococcus aureus. Presentation of cellulitis of the breast includes pain, reddening, and warmth to the touch; treatment includes antibiotics and/or aspiration.

Ideally, when should prenatal care begin? a. Before the first missed menstrual period b. After the first missed menstrual period c. After the second missed menstrual period d. After the third missed menstrual period

ANS: B Prenatal care should begin soon after the first missed menstrual period. This offers the greatest opportunities to ensure the health of the expectant mother and her infant. Prenatal care before missing the first menstrual period is too early. It is unlikely the woman is even aware of the pregnancy. Ideally, prenatal visits should begin soon after the first period is missed. Beginning prenatal care after the third missed menstrual period is too late. The woman will have completed the first trimester by that time.

The client has undergone hysteroscopic uterine ablation to destroy several smaller fibroids. The nurse is preparing to provide discharge instructions. Which information is a priority for the patient and should be included in the teaching plan? (Select all that apply.) a. Analgesics and nonsteroidal antiinflammatory drugs can be used for pain control. b. Vaginal discharge is to be expected for 5 to 7 days. c. Sexual activity can be resumed after 48 hours. d. Next menstrual period will be irregular. e. Provider should be notified if heavy bleeding occurs

ANS: A, D, E Before discharging the client, the following information should be given: analgesics can be used for pain relief as needed, normal activities can be resumed within several days, vaginal discharge is to be expected for 4 to 6 weeks, and the use of tampons and sexual activity should be avoided for 2 weeks. The next menstrual period will be irregular, and the provider should be notified for heavy bleeding or signs of infection.

Which client should the nurse refer for further testing? a. Left breast slightly smaller than right breast b. Eversion (elevation) of both nipples c. Faintly visible bilateral symmetry of venous network d. Small dimple located in the upper outer quadrant of the right breast

ANS: D A small dimple is an abnormal finding and should be further evaluated. Nipple retraction and a dimpling or pitting of the skin is suggestive of a locally advanced, aggressive form of breast cancer. In many women, one breast is smaller than the other, and eversion of both nipples is a normal finding. Faintly visible venous network is also a normal finding.

While assessing the vital signs of a pregnant woman in her third trimester, the client reports feeling faint, dizzy, and agitated. Which nursing intervention is appropriate? a. Have the patient stand up, and then retake her BP. b. Have the patient sit down, and then hold her arm in a dependent position. c. Have the patient lie supine for 5 minutes, and then recheck her BP on both arms. d. Have the patient turn to her left side, and then recheck her BP in 5 minutes.

ANS: D BP is affected by maternal position during pregnancy. The supine position may cause occlusion of the vena cava and descending aorta. Turning the pregnant woman to a lateral recumbent position alleviates pressure on the blood vessels and quickly corrects supine hypotension. Pressures are significantly higher when the client is standing. This option causes an increase in systolic and diastolic pressures. The arm should be supported at the same level of the heart. The supine position may cause occlusion of the vena cava and descending aorta, creating hypotension.

What is the importance of obtaining informed consent when educating a client regarding contraceptive methods? a. Contraception is an invasive procedure that requires hospitalization. b. The method may require a surgical procedure to insert a device. c. The contraception method chosen may be unreliable. d. The method chosen has potentially dangerous side effects.

ANS: D Being aware of the potential side effects is important for couples who are making an informed decision about the use of contraceptives. The only contraceptive method that is a surgical procedure and requires hospitalization is sterilization. Some methods have greater efficacy than others, and this efficacy should be included in the teaching.

Which statement regarding gamete intrafallopian transfer (GIFT) is most accurate? a. Semen is collected after laparoscopy. b. Women must have two normal fallopian tubes. c. Ovulation spontaneously occurs. d. Ova and sperm are transferred to one tube

ANS: D Similar to in vitro fertilization (IVF), GIFT requires the woman to have at least one normal tube. Ovulation is induced, and the oocytes are aspirated during laparoscopy. Semen is collected before laparoscopy. The ova and sperm are then transferred to one uterine tube, permitting natural fertilization and cleavage.

A nurse is providing education to a support group of women newly diagnosed with breast cancer. It is important for the nurse to discuss which factor related to breast cancer with the group? a. Genetic mutations account for 50% of women who will develop breast cancer. b. Breast cancer is the leading cause of cancer death in women. c. In the United States, 1 in 10 women will develop breast cancer in her lifetime. d. The exact cause of breast cancer remains unknown.

ANS: D The exact cause of breast cancer is unknown. 15% of these cancers are related to genetic mutations. Breast cancer is the second leading cause of cancer death in woman ages 45 to 55 years. One in eight women in the United States will develop breast cancer in her lifetime.

Kidney infection or stones

Severe backache and flank pain

Fetal jeopardy or intrauterine fetal death

Decreased fetal movement

. Gestational diabetes

Glycosuria

Stage I

Tumor less than 2 cm with negative nodes

Stage IIIb

Tumor of any size with extension to chest wall

Which statement regarding the probable signs of pregnancy is most accurate? a. Determined by ultrasound b. Observed by the health care provider c. Reported by the client d. Confirmed by diagnostic tests

ANS B Probable signs are those detected through trained examination. Fetal visualization is a positive sign of pregnancy. Presumptive signs are those reported by the client. The term diagnostic tests is open for interpretation. To diagnose pregnancy, one would have to see positive signs of pregnancy.

The practice of the calendar rhythm method is based on the number of days in each menstrual cycle. The fertile period is determined after monitoring each cycle for 6 months. The beginning of the fertile period is estimated by subtracting 18 days from the longest cycle and 11 days from the shortest. If the woman's cycles vary in length from 24 to 30 days, then her fertile period would be day _____ through day ______.

ANS: 6; 19 To avoid pregnancy, the couple must abstain from intercourse on days 6 through 19. Ovulation occurs on day 12 (plus or minus 2 days either way).

Which condition is the most common malignancy of the reproductive system? a. Endometrial cancer b. Cervical cancer c. Ovarian cancer d. Vulvar and vaginal cancer

ANS: A Endometrial cancer occurs most frequently in Caucasian women and after menopause. Certain viral infections and sexually transmitted diseases (STIs) create risks for cervical cancer. Ovarian cancer is the most malignant reproductive system cancer and accounts for the most deaths. Cancers of the vulva and vagina are relatively rare.

What is the correct term used to describe the mucous plug that forms in the endocervical canal? a. Operculum b. Leukorrhea c. Funic souffle d. Ballottement

ANS: A The operculum protects against bacterial invasion. Leukorrhea is the mucus that forms the endocervical plug (the operculum). The funic souffle is the sound of blood flowing through the umbilical vessels. Ballottement is a technique for palpating the fetus.

Researchers have found a number of common risk factors that increase a woman's chance of developing a breast malignancy. It is essential for the nurse who provides care to women of any age to be aware of which risk factors? (Select all that apply.) a. Family history b. Late menarche c. Early menopause d. Race e. Nulliparity or first pregnancy after age 40 years

ANS: A, D, E Family history, race, and nulliparity or the first pregnancy after age 40 years are known risk factors for the development of breast cancer. Others risk factors include age, personal history of cancer, high socioeconomic status, sedentary lifestyle, hormone replacement therapy, recent use of oral contraceptives, never having breastfed a child, and drinking more than one alcoholic beverage per day. Early menarche and late menopause are not risk factors for breast malignancy.

Which sign of a potential complication is the most important for the nurse to share with the client? a. Constipation b. Alteration in the pattern of fetal movement c. Heart palpitations d. Edema in the ankles and feet at the end of the day

ANS: B An alteration in the pattern or amount of fetal movement may indicate fetal jeopardy. Constipation, heart palpitations, and ankle and foot edema are normal discomforts of pregnancy that occur in the second and third trimesters.

Which statement best describes the rationale for the physiologic anemia that occurs during pregnancy? a. Physiologic anemia involves an inadequate intake of iron. b. Dilution of hemoglobin concentration occurs in pregnancy with physiologic anemia. c. Fetus establishes the iron stores. d. Decreased production of erythrocytes occur.

ANS: B When blood volume expansion is more pronounced and occurs earlier than the increase in red blood cells, the woman has physiologic anemia, which is the result of the dilution of hemoglobin concentration rather than inadequate hemoglobin. An inadequate intake of iron may lead to true anemia. The production of erythrocytes increases during pregnancy.

After a mastectomy, which activity should the client be instructed to avoid? a. Emptying surgical drains twice a day and as needed b. Lifting more than 4.5 kg (10 lb) or reaching above her head until given permission by her surgeon c. Wearing clothing with snug sleeves to support the tissue of the arm on the operative side d. Immediately reporting inflammation that develops at the incision site or in the affected arm

ANS: C The woman should not be advised to wear snug clothing. She should be advised to avoid tight clothing, tight jewelry, and other apparel that might cause decreased circulation in the affected arm. As part of the teaching plan, the woman should be instructed to empty the surgical drains twice a day, to avoid lifting more than 4.5 kg (10 lb) or reaching above her head until given permission by her surgeon, and to report immediately any inflammation that develops at the incision site or in the affected arm.

Male fertility declines slowly after age 40 years; however, no cessation of sperm production analogous to menopause in women occurs in men. What condition is not associated with advanced paternal age? a. Autosomal dominant disorder b. Schizophrenia c. Autism spectrum disorder d. Down syndrome

ANS: D Paternal age older than 40 years is associated with an increased risk for autosomal dominant disorder, schizophrenia, and autism spectrum disorder in their offspring. Although Down syndrome can occur in any pregnancy, it is often associated with advanced maternal age.

Hypertension, preeclampsia

Epigastric pain in late pregnancy

Stage IIb

Tumor 2 to 5 cm with positive node

A woman has preinvasive cancer of the cervix. Which modality would the nurse discuss as an available option for a client with this condition? a. Cryosurgery b. Colposcopy c. Hysterectomy d. Internal radiation

ANS: A Cryosurgery, laser surgery, and loop electrosurgical excision procedure (LEEP) are several techniques used to treat preinvasive lesions. Colposcopy is the examination of the cervix with a stereoscopic binocular microscope that magnifies the view of the cervix. This examination would have already been performed as part of the diagnosis of preinvasive cancer of the cervix. A hysterectomy is performed if the cancer has extended beyond the cervix. Women with positive pelvic nodes (indicating invasive cancer) usually receive whole pelvis irradiation.

Which is the most common technique used for the termination of a pregnancy in the second trimester? a. Dilation and evacuation (D&E) b. Methotrexate administration c. Prostaglandin administration d. Vacuum aspiration

ANS: A D&E can be performed at any point up to 20 weeks of gestation. It is more commonly performed between 13 and 16 weeks of gestation. Methotrexate is a cytotoxic drug that causes early abortion by preventing fetal cell division. Prostaglandins are also used for early abortion and work by dilating the cervix and initiating uterine wall contractions. Vacuum aspiration is used for abortions in the first trimester.

Which behavior indicates that a woman is "seeking safe passage" for herself and her infant? a. She keeps all prenatal appointments. b. She "eats for two." c. She slowly drives her car. d. She wears only low-heeled shoes

ANS: A The goal of prenatal care is to foster a safe birth for the infant and mother. Although properly eating, carefully driving, and using proper body mechanics all are healthy measures that a mother can take, obtaining prenatal care is the optimal method for providing safety for both herself and her baby.

The various systems and organs of the fetus develop at different stages. Which statement is most accurate? a. Cardiovascular system is the first organ system to function in the developing human. b. Hematopoiesis originating in the yolk sac begins in the liver at 10 weeks of gestation. c. Body changes from straight to C-shape occurs at 8 weeks of gestation. d. Gastrointestinal system is mature at 32 weeks of gestation.

ANS: A The heart is developmentally complete by the end of the embryonic stage. Hematopoiesis begins in the liver during the sixth week. The body becomes C-shaped at 21 weeks of gestation. The gastrointestinal system is complete at 36 weeks of gestation.

What represents a typical progression through the phases of a woman's establishing a relationship with the fetus? a. Accepts the fetus as distinct from herself—accepts the biologic fact of pregnancy—has feelings of caring and responsibility. b. Fantasizes about the child's gender and personality—views the child as part of herself—becomes introspective. c. Views the child as part of herself—has feelings of well-being—accepts the biologic fact of the pregnancy. d. "I am pregnant"—"I am going to have a baby"—"I am going to be a mother."

ANS D The woman first centers on herself as pregnant, then on the baby as an entity separate from herself, and then on her responsibilities as a mother. The expressions "I am pregnant," "I am going to have a baby," and "I am going to be a mother" sum up the progression through the three phases. In phase one, the woman views the child as part of herself and not as a separate being. This is only the first step of the progression through phases of attachment. Accepting the fetus as distinct from herself occurs during the second phase of emotional attachment. Fantasizing about the child's sex and personality based on fetal activity occurs during the third phase of attachment.

Which contraceptive method should the nurse identify as protecting against sexually transmitted infections (STIs) and the human immunodeficiency virus (HIV)? a. Periodic abstinence b. Barrier methods c. Hormonal methods d. Same protection with all methods

ANS: B Barrier methods, such as condoms, protect against STIs and the HIV the best of all contraceptive methods. Periodic abstinence and hormonal methods, such as birth control pills, offer no protection against STIs or the HIV.

The number of routine laboratory tests during follow-up visits is limited; however, those that are performed are essential. Which statements regarding group B Streptococcus (GBS) testing are correct? (Select all that apply.) a. Performed between 32 and 34 weeks of gestation. b. Performed between 35 and 37 weeks of gestation. c. All women should be tested. d. Only women planning a vaginal birth should be tested. e. Women with a history of GBS should be retested.

ANS: B, C, E GBS testing is recommended between 35 and 37 weeks of gestation; cultures collected earlier will not accurately predict the presence of GBS at birth. All women should be tested, even those planning an elective cesarean birth. Membranes may rupture early, requiring prophylactic antibiotics. Clients with a history of GBS should be retested.

While educating the client regarding the risks and benefits of a vasectomy, which information should the nurse include? (Select all that apply.) a. Sterilization should be performed under general anesthesia. b. Pain, bleeding, and infection are possible complications. c. For several months, pregnancy may still be possible. d. Vasectomy may affect potency. e. Secondary sex characteristics are unaffected.

ANS: B, C, E Vasectomy is the most commonly used procedure for male sterilization and is performed on an outpatient basis under local anesthesia. Pain, bleeding, swelling, and infection are considered complications. Reversal is generally unsuccessful; however, it may take several weeks to months for all sperm to be cleared from the sperm ducts. Another form of contraception is necessary until the sperm counts are zero. Vasectomy has no effect on potency, and secondary sex characteristics are not affected.

Which diagnostic test is used to confirm a suspected diagnosis of breast cancer? a. Mammogram b. Ultrasound c. Core needle biopsy d. Magnetic resonance imaging (MRI)

ANS: C When a suspicious mammogram is noted or a lump is detected, diagnosis is confirmed by either a core-needle biopsy or a needle-localization biopsy. Mammography is a clinical screening tool that may aid in the early detection of breast cancers. Transillumination, thermography, and ultrasound breast imaging are being explored as methods for detecting early breast carcinoma. An MRI is useful in women with masses that are difficult to find (occult breast cancer).

A pregnant woman at 25 weeks of gestation tells the nurse that she dropped a pan last week and her baby jumped at the noise. Which response by the nurse is most accurate? a. "That must have been a coincidence; babies can't respond like that." b. "The fetus is demonstrating the aural reflex." c. "Babies respond to sound starting at approximately 24 weeks of gestation." d. "Let me know if it happens again; we need to report that to your midwife."

ANS: C Babies respond to external sound starting at approximately 24 weeks of gestation. Acoustic stimulations can evoke a fetal heart rate response. There is no such thing as an aural reflex. The last statement is inappropriate and may cause undue psychologic alarm to the client.

A woman who is 14 weeks pregnant tells the nurse that she always had a glass of wine with dinner before she became pregnant. She has abstained during her first trimester and would like to know if it is safe for her to have a drink with dinner now. Which guidance should the nurse provide? a. "Since you're in your second trimester, there's no problem with having one drink with dinner." b. "One drink every night is too much. One drink three times a week should be fine." c. "Since you're in your second trimester, you can drink as much as you like." d. "Because no one knows how much or how little alcohol it takes to cause fetal problems, the best course is to abstain throughout your pregnancy."

ANS: D The statement "Because no one knows how much or how little alcohol it takes to cause fetal problems, the best course is to abstain throughout your pregnancy" is accurate. A safe level of alcohol consumption during pregnancy has not yet been established. Although the consumption of occasional alcoholic beverages may not be harmful to the mother or her developing fetus, complete abstinence is strongly advised.

A woman arrives at the clinic for a pregnancy test. The first day of her LMP was September 10, 2014. Her expected date of birth (EDB) is

ANS: June 17, 2015 Using the Nägele's rule, June 17, 2015, is the correct EDB. The EDB is calculated by subtracting 3 months from the first day of the LMP and adding 7 days + 1 year to the day of the LMP. Therefore, with an LMP of September 10, 2014: September 10, 2014 - 3 months = June 10, 2014 + 7 days = June 17, 2014 + 1 year = June 17, 2015.

What type of cultural concern is the most likely deterrent to many women seeking prenatal care? a. Religion b. Modesty c. Ignorance d. Belief that physicians are evil

ANS B A concern for modesty is a deterrent to many women seeking prenatal care. For some women, exposing body parts, especially to a man, is considered a major violation of their modesty. Many cultural variations are found in prenatal care. Even if the prenatal care described is familiar to a woman, some practices may conflict with the beliefs and practices of a subculture group to which she belongs.

A 3-year-old girl's mother is 6 months pregnant. What concern is this child most likely to verbalize? a. How the baby will get out? b. How will the baby eat? c. Will you die having the baby? d. What color eyes will the baby have?

ANS B By age 3 or 4 years, children like to be told the story of their own beginning and accept it being compared with the present pregnancy. They like to listen to the fetal heartbeat and feel the baby move. Sometimes they worry about how the baby is being fed and what it will wear. School-age children take a more clinical interest in their mother's pregnancy and may want to know "How did the baby get in there?" and "How will it get out?" Whether the child's mother will die does not tend to be the focus of her questions about the impending birth of a sibling. The baby's eye color does not tend to be the focus of children's questions about the impending birth of a sibling.

A woman who is 16 weeks pregnant has come in for a follow-up visit with her partner. To reassure the client regarding fetal well-being, which is the highest priority action for the nurse to perform? a. Assess the fetal heart tones with a Doppler stethoscope. b. Measure and document the girth of the woman's abdomen. c. Complete an ultrasound examination (sonogram) in a timely manner. d. Offer the woman and her partner the opportunity to listen to the fetal heart tones

ANS D To provide the parents with the greatest sense of reassurance, the nurse should offer to have the client and her significant other the chance to listen to their baby's heartbeat. A fetoscope can detect the fetal heart rate around 20 weeks of gestation. Doppler can detect the fetal heart rate between 10 and 12 weeks and should be performed as part of routine fetal assessment. Abdominal girth is not a valid measure for determining fetal well-being. Fundal height is an important measure that should be determined with precision, with the same technique and positioning of the client consistently used at every prenatal visit. Routine ultrasound examinations are recommended in early pregnancy; they date the pregnancy and provide useful information about the health of the fetus. However, they are not necessary at each prenatal visit.

Numerous changes in the integumentary system occur during pregnancy. Which change persists after birth? a. Epulis b. Chloasma c. Telangiectasia d. Striae gravidarum

ANS D Striae gravidarum, or stretch marks, reflect a separation within the underlying connective tissue of the skin. They usually fade after birth, although they never completely disappear. An epulis is a red, raised nodule on the gums that easily bleeds; it disappears or shrinks after giving birth. Chloasma, or the mask of pregnancy, is a blotchy, brown hyperpigmentation of the skin over the cheeks, nose, and forehead, especially in dark-complexioned pregnant women. Chloasma usually fades after the birth. Telangiectasia, or vascular spiders, are tiny, star-shaped or branchlike, slightly raised, pulsating end-arterioles usually found on the neck, thorax, face, and arms. They occur because of elevated levels of circulating estrogen and usually disappear after birth.

What kind of fetal anomalies are most often associated with oligohydramnios? a. Renal b. Cardiac c. Gastrointestinal d. Neurologic

ANS: A An amniotic fluid volume of less than 300 ml (oligohydramnios) is often associated with fetal renal anomalies. The amniotic fluid volume has no bearing on the fetal cardiovascular system. Gastrointestinal anomalies are associated with hydramnios or an amniotic fluid volume greater than 2 L. The amniotic fluid volume has no bearing on the fetal neurologic system.

What is the primary role of the doula during labor? a. Helps the woman perform breathing techniques and provides support to the woman and her partner b. Checks the fetal monitor tracing for effects of the labor process on the fetal heart rate c. Takes the place of the father as a coach and support provider d. Administers pain medications as needed by the woman

ANS: A A doula is professionally trained to provide labor support, including physical, emotional, and informational support, to both the woman and her partner during labor and the birth. The doula does not become involved with clinical tasks. While the doula provides support, the goal is not to take the place of anyone in the mother's support group.

What is important for the nurse to recognize regarding the new father and his acceptance of the pregnancy and preparation for childbirth? a. The father goes through three phases of acceptance of his own. b. The father's attachment to the fetus cannot be as strong as that of the mother because it does not start until after the birth. c. In the last 2 months of pregnancy, most expectant fathers suddenly get very protective of their established lifestyle and resist making changes to the home. d. Typically, men remain ambivalent about fatherhood right up to the birth of their child.

ANS: A A father typically goes through three phases of development to reach acceptance of fatherhood: the announcement phase, the moratorium phase, and the focusing phase. The father-child attachment can be as strong as the mother-child relationship and can also begin during pregnancy. During the last 2 months of the pregnancy, many expectant fathers work hard to improve the environment of the home for the child. Typically, the expectant father's ambivalence ends by the first trimester, and he progresses to adjusting to the reality of the situation and then to focusing on his rolE

Which explanation best describes a skin-sparing mastectomy? a. Removal of the breast, nipple, and areola, leaving the skin in tact b. Removal of the breast, nipple, areola, and axillary node dissection c. Incision on the outside of the breast, leaving the nipple intact d. Removal of both breasts in their entirety

ANS: A A skin-sparing mastectomy is a special procedure that keeps the outer breast of the skin intact. The breast, nipple, and areola are removed. A tissue expander may be placed for later reconstruction. None of the remaining options accurately describe this type of mastectomy.

A couple have arrived for their preprocedural In vitro fertilization-embryo transfer (IVF-ET) interview. Which explanation regarding the procedure is most accurate? a. "The procedure begins with collecting eggs from the woman's ovaries." b. "A donor embryo will be transferred into the woman's uterus." c. "Donor sperm will be used to inseminate the woman." d. "Don't worry about the technical stuff; that's what we are here for."

ANS: A A woman's eggs are collected from her ovaries, fertilized in the laboratory with the partner's sperm, and transferred to her uterus after normal embryonic development has occurred. Transferring a donor embryo to the woman's uterus describes the procedure for a donor embryo. Inseminating the woman with donor sperm describes therapeutic donor insemination. Telling the client not to worry discredits the client's need for teaching and is not the most appropriate response.

Which presumptive sign or symptom of pregnancy would a client experience who is approximately 10 weeks of gestation? a. Amenorrhea b. Positive pregnancy test c. Chadwick sign d. Hegar sign

ANS: A Amenorrhea is a presumptive sign of pregnancy. Presumptive signs of pregnancy are those felt by the woman. A positive pregnancy test and the presence of the Chadwick and Hegar signs are all probable signs of pregnancy.

Which information regarding amniotic fluid is important for the nurse to understand? a. Amniotic fluid serves as a source of oral fluid and a repository for waste from the fetus. b. Volume of the amniotic fluid remains approximately the same throughout the term of a healthy pregnancy. c. The study of fetal cells in amniotic fluid yields little information. d. A volume of more than 2 L of amniotic fluid is associated with fetal renal abnormalities.

ANS: A Amniotic fluid serves as a source of oral fluid, serves as a repository for waste from the fetus, cushions the fetus, and helps maintain a constant body temperature. The volume of amniotic fluid constantly changes. The study of amniotic fluid yields information regarding the sex of the fetus and the number of chromosomes. Too much amniotic fluid (hydramnios) is associated with gastrointestinal and other abnormalities.

Which women should undergo prenatal testing for the human immunodeficiency virus (HIV)? a. All women, regardless of risk factors b. Women who have had more than one sexual partner c. Women who have had a sexually transmitted infection (STI) d. Woman who are monogamous with one partner

ANS: A An HIV test is recommended for all women, regardless of risk factors. The incidence of perinatal transmission from an HIV-positive mother to her fetus ranges from 25% to 35%. Women who test positive for HIV can then be treated.

A woman in labor passes some thick meconium as her amniotic fluid ruptures. The client asks the nurse where the baby makes the meconium. What is the correct response by the nurse? a. Fetal intestines b. Fetal kidneys c. Amniotic fluid d. Placenta

ANS: A As the fetus nears term, fetal waste products accumulate in the intestines as dark green-to-black, tarry meconium. Meconium is not produced by the fetal kidneys nor should it be present in the amniotic fluid, which may be an indication of fetal compromise. The placenta does not produce meconium.

With regard to medications, herbs, boosters, and other substances normally encountered by pregnant women, what is important for the nurse to be aware of? a. Both prescription and over-the-counter (OTC) drugs that otherwise are harmless can be made hazardous by metabolic deficiencies of the fetus. b. The greatest danger of drug-caused developmental deficits in the fetus is observed in the final trimester. c. Killed-virus vaccines (e.g., tetanus) should not be administered during pregnancy, but live-virus vaccines (e.g., measles) are permissible. d. No convincing evidence exists that secondhand smoke is potentially dangerous to the fetus.

ANS: A Both prescription and OTC drugs that otherwise are harmless can be made hazardous by metabolic deficiencies of the fetus. This is especially true for new medications and combinations of drugs. The greatest danger of drug-caused developmental defects exists in the interval from fertilization through the first trimester, during which a woman may not realize that she is pregnant. Live-virus vaccines should be part of postpartum care; killed-virus vaccines may be administered during pregnancy. Secondhand smoke is associated with fetal growth restriction and increases in infant mortality.

A client has been prescribed adjuvant tamoxifen therapy. What common side effect might she experience? a. Weight gain, hot flashes, and blood clots b. Vomiting, weight loss, and hair loss c. Nausea, vomiting, and diarrhea d. Hot flashes, weight gain, and headaches

ANS: A Common side effects of tamoxifen therapy include hot flashes, weight gain, and blood clots. Weight loss, hair loss, diarrhea, and headaches are not common side effects of tamoxifen

A woman is in her seventh month of pregnancy. She reports episodes of nasal congestion and occasional epistaxis. Which statement best describes why this may be happening to this client? a. This respiratory change is normal in pregnancy and caused by an elevated level of estrogen. b. This cardiovascular change is abnormal, and the nosebleeds are an ominous sign. c. The woman is a victim of domestic violence and is being hit in the face by her partner. d. The woman has been likely intranasally using cocaine for several months.

ANS: A Elevated levels of estrogen cause capillaries to become engorged in the respiratory tract, which may result in edema in the nose, larynx, trachea, and bronchi. This congestion may cause nasal stuffiness and epistaxis. Cardiovascular changes in pregnancy may cause edema in the lower extremities. Domestic violence cannot be determined on the basis on the sparse facts provided. If the woman had been hit in the face, then she most likely would have additional physical findings. Cocaine use cannot be determined on the basis on the sparse facts provided.

Which statement regarding emergency contraception is correct? a. Emergency contraception requires that the first dose be taken within 120 hours of unprotected intercourse. b. Emergency contraception may be taken right after ovulation. c. Emergency contraception has an effectiveness rate in preventing pregnancy of approximately 50%. d. Emergency contraception is commonly associated with the side effect of menorrhagia.

ANS: A Emergency contraception should be taken as soon as possible or within 72 hours of unprotected intercourse to prevent pregnancy. If taken before ovulation, follicular development is inhibited, which prevents ovulation. The risk of pregnancy is reduced by as much as 75%. The most common side effect of postcoital contraception is nausea.

Which woman has the highest risk for endometrial cancer? a. Postmenopausal woman with hypertension b. Woman who has an intrauterine device (IUD) c. Client who has been on birth control for 15 years d. Perimenopausal woman who has a cystocele

ANS: A Endometrial cancer is most often seen in postmenopausal women between the ages of 50 and 65 years. Hypertension is a risk factor associated with the development of this malignancy. The use of an IUD does not increase a woman's risk for endometrial cancer. A client who has been on birth control for 15 years is not at increased risk for endometrial cancer; the birth control contraceptives might offer some protection. The development of a cystocele will not increase a woman's risk for endometrial cancer

A woman asks the nurse, "What protects my baby's umbilical cord from being squashed while the baby's inside of me?" What is the nurse's best response? a. "Your baby's umbilical cord is surrounded by connective tissue called Wharton's jelly, which prevents compression of the blood vessels." b. "Your baby's umbilical cord floats around in blood and amniotic fluid." c. "You don't need to be worrying about things like that." d. "The umbilical cord is a group of blood vessels that are very well protected by the placenta."

ANS: A Explaining the structure and function of the umbilical cord is the most appropriate response. Connective tissue called Wharton's jelly surrounds the umbilical cord, prevents compression of the blood vessels, and ensures continued nourishment of the embryo or fetus. The umbilical cord does not float around in blood or fluid. Telling the client not to worry negates her need for information and discounts her feelings. The placenta does not protect the umbilical cord.

Which term best describes the conscious decision concerning deciding when to conceive or avoid pregnancy as opposed to the intentional prevention of pregnancy during intercourse? a. Family planning b. Birth control c. Contraception d. Assisted reproductive therapy

ANS: A Family planning is the process of deciding when and if to have children. Birth control is the device and/or practice used to reduce the risk of conceiving or bearing children. Contraception is the intentional prevention of pregnancy during sexual intercourse. Assisted reproductive therapy is one of several possible treatments for infertility.

What is the correct name describing a benign breast condition that includes dilation and inflammation of the collecting ducts? a. Mammary duct ectasia b. Intraductal papilloma c. Chronic cystic disease d. Fibroadenoma

ANS: A Generally occurring in women approaching menopause, mammary duct ectasia results in a firm irregular mass in the breast, enlarged axillary nodes, and nipple discharge. Intraductal papillomas develop in the epithelium of the ducts of the breasts; as the mass grows, it causes trauma or erosion within the ducts. Chronic cystic disease causes pain and tenderness. The cysts that form are multiple, smooth, and well delineated. Fibroadenoma is evidenced by fibrous and glandular tissues. They are felt as firm, rubbery, and freely mobile nodules

A pregnant client tells her nurse that she is worried about the blotchy, brownish coloring over her cheeks, nose, and forehead. The nurse can reassure her that this is a normal condition related to hormonal changes. What is the correct term for this integumentary finding? a. Melasma b. Linea nigra c. Striae gravidarum d. Palmar erythema

ANS: A Melasma, (also called chloasma, the mask of pregnancy), usually fades after birth. This hyperpigmentation of the skin is more common in women with a dark complexion. Melasma appears in 50% to 70% of pregnant women. Linea nigra is a pigmented line that runs vertically up the abdomen. Striae gravidarum are also known as stretch marks. Palmar erythema is signified by pinkish red blotches on the hands.

A postmenopausal woman has been diagnosed with two leiomyomas (fibroids). Which clinical finding is most commonly associated with the presence of leiomyomas? a. Abnormal uterine bleeding b. Diarrhea c. Weight loss d. Acute abdominal pain

ANS: A Most women are asymptomatic. Abnormal uterine bleeding is the most common symptom of leiomyomas. Diarrhea is not commonly associated with leiomyomas. Weight loss does not usually occur in the woman with leiomyomas, and the client with leiomyomas is unlikely to experience abdominal pain.

The nurse caring for a pregnant client is evaluating health teaching regarding fetal circulation. Which statement from the client reassures the nurse that the teaching has been effective? a. "Optimal fetal circulation is achieved when I am in the side-lying position." b. "Optimal fetal circulation is achieved when I am on my back with a pillow under my knees." c. "Optimal fetal circulation is achieved when the head of the bed is elevated." d. "Optimal fetal circulation is achieved when I am on my abdomen."

ANS: A Optimal circulation is achieved when the woman is lying at rest on her side. Decreased uterine circulation may lead to intrauterine growth restriction. Previously, it was believed that the left lateral position promoted maternal cardiac output, enhancing blood flow to the fetus. However, it is now known that the side-lying position enhances uteroplacental blood flow. If a woman lies on her back with the pressure of the uterus compressing the vena cava, then blood return to the right atrium is diminished. Although having the head of the bed elevated is recommended and ideal for later in pregnancy, the woman still must maintain a lateral tilt to the pelvis to avoid compressing the vena cava. Many women find lying on their abdomen uncomfortable as pregnancy advances. Side-lying is the ideal position to promote blood flow to the fetus.

A new mother asks the nurse about the "white substance" covering her infant. How should the nurse explain the purpose of vernix caseosa? a. Vernix caseosa protects the fetal skin from the amniotic fluid. b. Vernix caseosa promotes the normal development of the peripheral nervous system. c. Vernix caseosa allows the transport of oxygen and nutrients across the amnion. d. Vernix caseosa regulates fetal temperature.

ANS: A Prolonged exposure to the amniotic fluid during the fetal period could result in the breakdown of the skin without the protection of the vernix caseosa. Normal development of the peripheral nervous system was dependent on nutritional intake of the mother. The amnion was the inner membrane that surrounded the fetus and was not involved in the oxygen and nutrient exchange. The amniotic fluid helped maintain fetal temperature.

The most conservative approach for early breast cancer treatment involves lumpectomy followed by which procedure? a. Radiation b. Adjuvant systemic therapy c. Hormonal therapy d. Chemotherapy

ANS: A Radiation therapy, in the form of either brachytherapy or accelerated breast radiation, is the standard therapy after lumpectomy for the treatment of early-stage breast cancer. Chemotherapy administered soon after surgical removal of the tumor is referred to as adjuvant chemotherapy. Not all women are candidates for hormonal therapy. After the entire tumor or portion is removed by excision, a receptor assay must be performed. Chemotherapy with multiple-drug combinations is used in the treatment of recurrent and advanced breast cancer with positive results.

A first-time mother at 18 weeks of gestation is in for her regularly scheduled prenatal visit. The client tells the nurse that she is afraid that she is going into premature labor because she is beginning to have regular contractions. The nurse explains that these are Braxton Hicks contractions. What other information is important for the nurse to share? a. Braxton Hicks contractions should be painless. b. They may increase in frequency with walking. c. These contractions might cause cervical dilation. d. Braxton Hicks contractions will impede oxygen flow to the fetus.

ANS: A Soon after the fourth month of gestation, uterine contractions can be felt through the abdominal wall. Braxton Hicks contractions are regular and painless and continue throughout the pregnancy. Although they are not painful, some women complain that they are annoying. This type of contraction usually ceases with walking or exercise. Braxton Hicks contractions can be mistaken for true labor; however, they do not increase in intensity, frequency, or cause cervical dilation. These contractions facilitate uterine blood flow through the intervillous spaces of the placenta and thereby promote oxygen delivery to the fetus.

What important, immediate postoperative care practice should the nurse remember when caring for a woman who has had a mastectomy? a. The blood pressure (BP) cuff should not be applied to the affected arm. b. Venipuncture for blood work should be performed on the affected arm. c. The affected arm should be used for intravenous (IV) therapy. d. The affected arm should be held down close to the woman's side.

ANS: A The affected arm should not be used for BP readings, IV therapy, or venipuncture. The affected arm should be elevated with pillows above the level of the right atrium.

Why might it be more difficult to diagnose appendicitis during pregnancy? a. The appendix is displaced upward and laterally, high and to the right. b. The appendix is displaced upward and laterally, high and to the left. c. The appendix is deep at the McBurney's point. d. The appendix is displaced downward and laterally, low and to the right

ANS: A The appendix is displaced high and to the right, not to the left. It is displaced beyond the McBurney's point and is not displaced in a downward direction.

The diagnosis of pregnancy is based on which positive signs of pregnancy? (Select all that apply.) a. Identification of fetal heartbeat b. Palpation of fetal outline c. Visualization of the fetus d. Verification of fetal movement e. Positive hCG test

ANS: A, C, D Identification of a fetal heartbeat, the visualization of the fetus, and verification of fetal movement are all positive, objective signs of pregnancy. Palpation of fetal outline and positive hCG test are probable signs of pregnancy. A tumor also can be palpated. Medication and tumors may lead to false-positive results on pregnancy tests.

A woman has chosen the calendar method of conception control. Which is the most important action the nurse should perform during the assessment process in preparation to discuss the implementation of this method? a. Obtain a history of the woman's menstrual cycle lengths for the past 6 to 12 months. b. Determine the client's weight gain and loss pattern for the previous year. c. Examine skin pigmentation and hair texture for hormonal changes. d. Explore the client's previous experiences with conception control.

ANS: A The calendar method of conception control is based on the number of days in each cycle, counting from the first day of menses. The fertile period is determined after the lengths of menstrual cycles have been accurately recorded for 6 months. Weight gain or loss may be partly related to hormonal fluctuations, but it has no bearing on the use of the calendar method. Integumentary changes may be related to hormonal changes, but they are not indicators for use of the calendar method. Exploring previous experiences with conception control may demonstrate client understanding and compliancy, but these experiences are not the most important aspect to assess for the discussion of the calendar method

Considering the treatment plan for a pregnant woman with gynecologic cancer, which statement about timing or type of treatment is correct? a. The fetus is most at risk during the first trimester. b. The fetus is most at risk during the second trimester. c. The fetus is most at risk during the third trimester. d. Surgery is riskier than chemotherapy in the first trimester.

ANS: A The first trimester is the most vulnerable period for the growing fetus. Women may be faced with deciding about terminating the pregnancy, depending on the stage and extent of the disease. For advanced disease in the second trimester, alkylating agents, 5-fluorouracil (5-FU), and vincristine are relatively safe for the fetus. For advanced disease in the third trimester, alkylating agents, 5-FU, and vincristine are relatively safe for the fetus. Surgery is less risky than chemotherapy in the first trimester.

Which comment by the nurse would be most helpful in instruct the woman on how best to reduce her risk of developing toxic shock syndrome (TSS)? (Select all that apply.) a. "You should always leave your diaphragm in place for at 6 hours after intercourse." b. "You should remove your diaphragm right after intercourse to prevent TSS." c. "You can use your diaphragm during your menstrual cycle as long as you clean it well." d. "Make sure you don't leave your diaphragm in for longer than 24 hours" e. Report a fever of 38.4 0 C to your health care provider immediately."

ANS: A The nurse should instruct the client on the proper use and removal of the diaphragm and include the danger signs of TSS. The diaphragm must remain against the cervix for 6 to 8 hours to prevent pregnancy, but it should not remain in place longer than 8 hours to avoid the risk of TSS. The diaphragm should not be used during menses. A fever of 38.4 0 C or higher could be a symptom of TSS and should be reported immediately.

. A client at 34 weeks of gestation seeks guidance from the nurse regarding personal hygiene. Which information should the nurse provide? a. Tub bathing is permitted even in late pregnancy unless membranes have ruptured. b. The perineum should be wiped from back to front. c. Bubble bath and bath oils are permissible because they add an extra soothing and cleansing action to the bath. d. Expectant mothers should use specially treated soap to cleanse the nipples.

ANS: A The primary danger from taking baths is falling in the tub. The perineum should be wiped from front to back. Bubble baths and bath oils should be avoided because they may irritate the urethra. Soap, alcohol, ointments, and tinctures should not be used to cleanse the nipples because they remove protective oils.

A woman who has undergone an induced abortion be instructed to return to the emergency department when what situation exists? (Select all that apply.) a. Fever higher than 38 C b. Chills c. Foul-smelling vaginal discharge d. Bleeding greater than 2 pads in 2 hours e. Abdominal tenderness and or pain

ANS: A, B, C, D, E The client should report to a health care facility for any of the following symptoms: fever higher than 38 C, chills, bleeding more than two saturated pads in 2 hours or heavy bleeding lasting for days, foul-smelling discharge, abdominal tenderness or pain, and cramping or backache.

The client is undergoing treatment for ovarian cancer. Which common nutritional problems are related to gynecologic cancers and the treatment thereof? (Select all that apply.) a. Stomatitis b. Constipation c. Increased appetite d. Diarrhea e. Nausea and vomiting

ANS: A, B, D, E Altered taste, stomatitis, constipation, anorexia, diarrhea, and nausea and vomiting are all possible nutritional complications related to gynecologic cancers and their treatment. The nurse must assess accordingly and adapt the client's plan of care. To ensure recovery, these women should consume a diet high in iron and protein, drink plenty of fluids, and eat foods high in vitamins C, B, and K.

A woman has just moved to the United States from Mexico. She is 3 months pregnant and has arrived for her first prenatal visit. During her assessment interview, the nurse learns that the client has not had any immunizations. Which immunizations should she receive at this point in her pregnancy? (Select all that apply.) a. Tetanus b. Diphtheria c. Chickenpox d. Rubella e. Hepatitis B

ANS: A, B, E Vaccines consisting of killed viruses may be used. Those that may be administered during pregnancy include tetanus, diphtheria, recombinant hepatitis B, and rabies vaccines. Immunizations with live or attenuated viruses are contraindicated during pregnancy because of their potential teratogenicity. Live-virus vaccines include those for measles (rubeola and rubella), chickenpox, and mumps.

Relating to the fetal circulatory system, which special characteristics allow the fetus to obtain enough oxygen from the maternal blood? (Select all that apply.) a. Fetal hemoglobin (Hb) carries 20% to 30% more oxygen than maternal Hb. b. Fetal Hb carries 40% to 50% more oxygen than maternal Hb. c. Hb concentration is 50% higher than that of the mother. d. Fetal heart rate is 110 to 160 beats per minute. e. Fetal heart rate is 160 to 200 beats per minute.

ANS: A, C, D The following three special characteristics enable the fetus to obtain sufficient oxygen from maternal blood: (1) the fetal Hb carries 20% to 30% more oxygen; (2) the concentration is 50% higher than that of the mother; and (3) the fetal heart rate is 110 to 160 beats per minute, a cardiac output that is higher than that of an adult.

A probable cause for increasing infertility is the societal delay in pregnancy until later in life. What are the natural reasons for the decrease in female fertility? (Select all that apply.) a. Ovulation dysfunction b. Endocrine dysfunction c. Organ damage from toxins d. Endometriosis e. Tubal infections

ANS: A, C, D, E All of these factors may result in a cumulative effect, decreasing fertility in women. Male infertility is more often caused by unfavorable sperm production attributable to endocrine dysfunction or cumulative metabolic disease.

Chemotherapy with multiple drug agents is used in the treatment of recurrent and advanced breast cancer with positive results. Which side effects would the nurse anticipate for the client once treatment has begun? (Select all that apply.) a. Hair loss b. Severe constipation c. Anemia d. Leukopenia e. Thrombocytopenia

ANS: A, C, D, E Because chemotherapeutic agents rapidly kill reproducing cells, treatment also affects normal cells that frequently reproduce. The side effects that the nurse would anticipate and on which the nurse will provide education include partial or full hair loss, gastrointestinal effects (e.g., nausea, vomiting, anorexia, mucositis), leukopenia, neutropenia, thrombocytopenia, and anemia.

The nurse is reviewing the educational packet provided to a client about tubal ligation. Which information regarding this procedure is important for the nurse to share? (Select all that apply.) a. "It is highly unlikely that you will become pregnant after the procedure." b. "Tubal ligation is an effective form of 100% permanent sterilization. You won't be able to get pregnant." c. "Sterilization offers some form of protection against sexually transmitted infections (STIs)." d. "Sterilization offers no protection against sexually transmitted infections (STIs)." e. "Your menstrual cycle will greatly increase after your sterilization."

ANS: A, D TEST BANK FOR MATERNITY AND WOMEN'S HEALTH CARE 12TH EDITION BY LOWDERMILK NURSINGKING.COM NURSINGKING.COM A woman is unlikely to become pregnant after tubal ligation. However, sterilization offers no protection against STIs and is not 100% effective. Typically, the menstrual cycle remains the same after a tubal ligation

The nurse should be aware that a pessary is most effective in the treatment of which disorder? a. Cystocele b. Uterine prolapse c. Rectocele d. Stress urinary incontinence

ANS: B A fitted pessary may be inserted into the vagina to support the uterus and hold it in the correct position. Usually a pessary is used for only a short time and is not used for the client with a cystocele. A rectocele cannot be corrected using a pessary. A pessary is not likely the most effective treatment for stress incontinence.

During the first trimester, which of the following changes regarding her sexual drive should a client be taught to expect? a. Increased sexual drive, because of enlarging breasts b. Decreased sexual drive, because of nausea and fatigue c. No change in her sexual drive d. Increased sexual drive, because of increased levels of female hormones

ANS: B A pregnant woman usually experiences a decrease, not an increase, in libido during the first trimester. Maternal physiologic changes, such as breast enlargement, nausea, fatigue, abdominal changes, perineal enlargement, leukorrhea, pelvic vasocongestion, and orgasmic responses, may affect sexuality and sexual expression. Libido may be depressed in the first trimester but often increases during the second and third trimesters. During pregnancy, the breasts may become enlarged and tender, which tends to interfere with coitus, thereby decreasing the desire to engage in sexual activity.

A woman who is 16 weeks pregnant asks the nurse, "Is it possible to tell by ultrasound if the baby is a boy or girl yet?" What is the best answer? a. "A baby's sex is determined as soon as conception occurs." b. "The baby has developed enough to enable us to determine the sex by examining the genitals through an ultrasound scan." c. "Boys and girls look alike until approximately 20 weeks after conception, and then they begin to look different." d. "It might be possible to determine your baby's sex, but the external organs look very similar right now."

ANS: B Although gender is determined at conception, the external genitalia of males and females look similar through the ninth week. By the twelfth week, the external genitalia are distinguishable as male or female.

The musculoskeletal system adapts to the changes that occur throughout the pregnancy. Which musculoskeletal alteration should the client expect? a. Her center of gravity will shift backward. b. She will have increased lordosis. c. She will have increased abdominal muscle tone. d. She will notice decreased mobility of her pelvic joints.

ANS: B An increase in the normal lumbosacral curve (lordosis) develops, and a compensatory curvature in the cervicodorsal region develops to help her maintain balance. The center of gravity shifts forward. She will have decreased abdominal muscle tone and will notice increased mobility of her pelvic joints.

A 31-year-old woman believes that she may be pregnant. She took an over-the-counter (OTC) pregnancy test 1 week ago after missing her period; the test was positive. During her assessment interview, the nurse asks about the woman's last menstrual period and whether she is taking any medications. The client states that she takes medicine for epilepsy. She has been under considerable stress lately at work and has not been sleeping well. Her physical examination does not indicate that she is pregnant. She has an ultrasound scan, which confirms that she is not pregnant. What is the most likely cause of the false-positive pregnancy test result? a. The pregnancy test was taken too early. b. Anticonvulsant medications may cause the false-positive test result. c. The woman has a fibroid tumor. d. She has been under considerable stress and has a hormone imbalance.

ANS: B Anticonvulsants may cause false-positive pregnancy test results. OTC pregnancy tests use enzyme-linked immunosorbent assay (ELISA) technology, which can yield positive results as soon as 4 days after implantation. Implantation occurs 6 to 10 days after conception. If the woman were pregnant, then she would be into her third week at this point (having missed her period 1 week ago). Fibroid tumors do not produce hormones and have no bearing on human chorionic gonadotropin (hCG) pregnancy tests. Although stress may interrupt normal hormone cycles (menstrual cycles), it does not affect hCG levels or produce positive pregnancy test results

Which statement regarding the structure and function of the placenta is correct? a. Produces nutrients for fetal nutrition b. Secretes both estrogen and progesterone c. Forms a protective, impenetrable barrier to microorganisms such as bacteria and viruses d. Excretes prolactin and insulin

ANS: B As one of its early functions, the placenta acts as an endocrine gland, producing four hormones necessary to maintain the pregnancy and to support the embryo or fetus: human chorionic gonadotropin (hCG), human placental lactogen (hPL), estrogen, and progesterone. The placenta does not produce nutrients. It functions as a means of metabolic exchange between the maternal and fetal blood supplies. Many bacteria and viruses can cross the placental membrane.

A woman who has a seizure disorder and takes barbiturates and phenytoin sodium daily asks the nurse about the pill as a contraceptive choice. What is the nurse's best response? a. "Oral contraceptives are a highly effective method, but they have some side effects." b. "Your current medications will reduce the effectiveness of the pill." c. "Oral contraceptives will reduce the effectiveness of your seizure medication." d. "The pill is a good choice for a woman of your age and with your personal history."

ANS: B Because the liver metabolizes oral contraceptives, their effectiveness is reduced when they are simultaneously taken with anticonvulsants. Stating that the pill is an effective birth control method with side effects is a true statement, but this response is not the most appropriate. The anticonvulsant reduces the effectiveness of the pill, not the other way around. Stating that the pill is a good choice for a woman of her age and personal history does not teach the client that the effectiveness of the pill may be reduced because of her anticonvulsant therapy.

Which renal system adaptation is an anticipated anatomic change of pregnancy? a. Increased urinary output makes pregnant women less susceptible to urinary infections. b. Increased bladder sensitivity and then compression of the bladder by the enlarging uterus result in the urge to urinate even when the bladder is almost empty. c. Renal (kidney) function is more efficient when the woman assumes a supine position. d. Using diuretic agents during pregnancy can help keep kidney function regular.

ANS: B Bladder sensitivity and then compression of the bladder by the uterus result in the urge to urinate more often, even when the bladder is almost empty. A number of anatomic changes in pregnancy make a woman more susceptible to urinary tract infections. Renal function is more efficient when the woman lies in the lateral recumbent position and is less efficient when she is supine. Diuretic use during pregnancy can overstress the system and cause problems.

Which ovarian neoplasm is described as a growth that contains hair, teeth, and sebaceous secretions? a. Ovarian fibroma b. Dermoid cyst c. Uterine polyp d. Follicular cyst

ANS: B Dermoid cysts are germ cell tumors, usually occurring in childhood, that may contain teeth, hair, bones, and sebaceous secretions and may unilaterally or bilaterally develop. Treatment is most often surgical removal. An ovarian fibroma is a solid ovarian neoplasm that develops from connective tissue, usually after menopause. A uterine polyp is a tumor that grows on the uterine wall on a stalk or pedicle. A follicular cyst develops within the ovaries of young women in response to follicle rupture and should resolve within one or two menstrual cycles

A client is concerned because she has been experiencing some milky, sticky breast discharge. Which nonmalignant condition is exhibited with this finding? a. Relative inflammatory lesion b. Galactorrhea c. Mammary duct ectasia d. Breast infection

ANS: B Galactorrhea bilaterally exhibits a spontaneous, milky, and sticky discharge and is a normal finding during pregnancy; however, it may also occur as the result of elevated prolactin levels. Prolactin can become elevated as a result of a thyroid disorder, pituitary tumor, stress, coitus, trauma, or chest wall surgery

Which statement concerning neurologic and sensory development in the fetus is correct? a. Brain waves have been recorded on an electroencephalogram as early as the end of the first trimester (12 weeks of gestation). b. Fetuses respond to sound by 24 weeks of gestation and can be soothed by the sound of the mother's voice. c. Eyes are first receptive to light at 34 to 36 weeks of gestation. d. At term, the fetal brain is at least one third the size of an adult brain.

ANS: B Hearing develops early and is fully developed at birth. Brain waves have been recorded at week 8. Eyes are receptive to light at 28 weeks of gestation. The fetal brain is approximately one fourth the size of an adult brain.

Breast hyperplasia can be corrected with a. Which statement regarding reduction mammoplasty is the most accurate? a. Breast reduction surgery is covered by insurance. b. Breastfeeding might be difficult. c. No sequelae after the procedure is known. d. Pain in the back and shoulders may not be relieved.

ANS: B If breast reduction surgery is performed, then establishing breastfeeding at a later date may be difficult. Macromastia may not be covered by all insurance companies. A consequence of surgery may be decreased sensation and pain, secondary to scar tissue. Reduction mammoplasty will relieve chronic neck and back pain.

A client is seen at the clinic at 14 weeks of gestation for a follow-up appointment. At which level does the nurse expect to palpate the fundus? a. Nonpalpable above the symphysis at 14 weeks of gestation b. Slightly above the symphysis pubis c. At the level of the umbilicus d. Slightly above the umbilicus

ANS: B In normal pregnancies, the uterus grows at a predictable rate. It may be palpated above the symphysis pubis sometime between the 12th and 14th weeks of pregnancy. As the uterus grows, it may be palpated above the symphysis pubis sometime between the 12th and 14th weeks of pregnancy. At 14 weeks, the uterus is not yet at the level of the umbilicus. The fundus is not palpable above the umbilicus until 22 to 24 weeks of gestation.

The pancreas forms in the foregut during the 5th to 8th week of gestation. A client with poorly controlled gestational diabetes asks the nurse what the effects of her condition will be on the fetus. What is the best response by the nurse? Poorly controlled maternal gestational diabetes will: a. produce fetal hypoglycemia. b. result in a macrocosmic fetus. c. result in a microcosmic fetus. d. enhance lung maturation.

ANS: B Insulin is produced by week 20 of gestation. In the fetus of a mother with uncontrolled diabetes, maternal hypoglycemia produces fetal hypoglycemia and macrocosmia results. Hyperinsulinemia blocks lung maturation, placing the neonate at risk for respiratory distress.

The nurse knows that teaching about the natural family planning method of contraception was effective when the couple responds that an ovum is considered fertile for which period of time? a. 6 to 8 hours b. 24 hours c. 2 to 3 days d. 1 week

ANS: B Most ova remain fertile for approximately 24 hours after ovulation, much longer than 6 to 8 hours. However, ova do not remain fertile for 2 to 3 days or are viable for 1 week. If unfertilized by a sperm after 24 hours, the ovum degenerates and is reabsorbed.

A patient in her first trimester complains of nausea and vomiting. She asks, "Why does this happen?" What is the nurse's best response? a. "Nausea and vomiting are due to an increase in gastric motility." b. "Nausea and vomiting may be due to changes in hormones." c. "Nausea and vomiting are related to an increase in glucose levels." d. "Nausea and vomiting are caused by a decrease in gastric secretions."

ANS: B Nausea and vomiting are believed to be caused by increased levels of hormones, decreased gastric motility, and hypoglycemia. Gastric motility decreases during pregnancy. Glucose levels decrease in the first trimester. Although gastric secretions decrease, these secretions are not the primary cause of the nausea and vomiting.

A male client asks the nurse why it is better to purchase condoms that are not lubricated with nonoxynol-9 (a common spermicide). Which response by the nurse is the most accurate? a. "The lubricant prevents vaginal irritation." b. "Nonoxynol-9 does not provide protection against STIs as originally thought" c. "The additional lubrication improves sex." d. "Nonoxynol-9 improves penile sensitivity."

ANS: B Nonoxynol-9 does not provide protection against STIs as originally thought; it has also been linked to an increase in the transmission of the HIV and can cause genital lesions. Nonoxynol-9 may cause vaginal irritation, has no effect on the quality of sexual activity, and has no effect on penile sensitivity.

Which of these interventions should the nurse instruct the client diagnosed with urinary incontinence (UI) to use first? a. Pelvic floor support devices b. Pelvic muscle exercises c. Surgery d. Medications

ANS: B Pelvic muscle exercises, known as Kegel exercises, along with bladder training can significantly decrease or entirely relieve stress incontinence in many women. Pelvic floor support devices, also known as pessaries, come in a variety of shapes and sizes. Pessaries may not be effective for all women and require scrupulous cleaning to prevent infection. Anterior and posterior repairs and even a hysterectomy may be performed. If surgical repair is performed, then the nurse must focus her care on preventing infection and helping the woman avoid putting stress on the surgical site. Pharmacologic therapy includes selective serotonin-norepinephrine reuptake inhibitors or vaginal estrogen therapy. However, pharmacologic therapy is not the first action a nurse should recommend.

A pregnant woman at 18 weeks of gestation calls the clinic to report that she has been experiencing occasional backaches of mild-to-moderate intensity. Which intervention should the nurse recommend? a. Kegel exercises b. Pelvic rock exercises c. Softer mattress d. Bed rest for 24 hours

ANS: B Pelvic rock exercises may help stretch and strengthen the abdominal and lower back muscles and relieve low back pain. Stretching and other exercises to relieve back pain should be performed several times a day. Kegel exercises increase the tone of the pelvic area, not the back. A softer mattress may not provide the support needed to maintain proper alignment of the spine and may contribute to back pain.

Significant advances have been made with most reproductive technologies. Which improvement has resulted in increased success related to preimplantation genetic diagnosis? a. Embryos are transferred at the cleavage stage. b. Embryos are transferred at the blastocyst stage. c. More than two embryos can be transferred at a time. d. Two cells are removed from each embryo.

ANS: B Preimplantation genetic diagnosis can be performed on a single cell removed from each embryo after 3 to 4 days. With the availability of extended culture mediums, embryos are transferred at the blastocyst stage (day 5), which increases the chance of a live birth, compared with the older practice of transferring embryos at the cleavage stage (day 3). No more than two embryos should be transferred at a time.

Which finding in the urinalysis of a pregnant woman is considered a variation of normal? a. Proteinuria b. Glycosuria c. Bacteria in the urine d. Ketones in the urine

ANS: B Small amounts of glucose may indicate physiologic spilling. The presence of protein could indicate kidney disease or preeclampsia. Urinary tract infections are associated with bacteria in the urine. An increase in ketones indicates that the patient is exercising too strenuously or has an inadequate fluid and food intake.

The client is instructed to place her thumb and forefinger on the areola and gently press inward. What is the purpose of this exercise? a. To check the sensitivity of the nipples b. To determine whether the nipple is everted or inverted c. To calculate the adipose buildup in the abdomen d. To see whether the fetus has become inactive

ANS: B Sometimes known as the pinch test, this exercise is used to determine whether the nipple is everted or inverted. Nipples must be everted to allow breastfeeding to occur naturally. The pinch does not determine the level of sensitivity of the nipples, nor is it not used to determine the level of adipose tissue in the abdomen. Fetal activity is not determined by using the pinch test.

A 48-year-old woman has just had a hysterectomy for endometrial cancer. Which statement alerts the nurse that further teaching is needed? a. "I can't wait to go on the cruise that I have planned for this summer." b. "I know that the surgery saved my life, but I will miss having sexual intercourse with my husband." c. "I have asked my daughter to come and stay with me next week after I am discharged from the hospital." d. "Well, I don't have to worry about getting pregnant anymore."

ANS: B Stating that she will miss having sexual intercourse with her husband indicates that further teaching is needed for this client regarding sexual activities after a hysterectomy. Although intercourse may be initially uncomfortable, the use of water-soluble lubricants, relaxation exercises, and changes in position may be helpful. Expressing plans for a vacation is a positive psychologic state with plans for the future. Stating that her daughter will stay with her indicates the client understands that she may need assistance during her acute recovery period. Stating that she no longer needs to worry about getting pregnant indicates knowledge related to the reproductive cycle and a positive outlook.

Which action should the nurse first take when meeting with a new client to discuss contraception? a. Obtain data about the frequency of coitus. b. Determine the woman's level of knowledge concerning contraception.. c. Assess the woman's willingness to touch her genitals and cervical mucus. d. Evaluate the woman's contraceptive life plan.

ANS: B TEST BANK FOR MATERNITY AND WOMEN'S HEALTH CARE 12TH EDITION BY LOWDERMILK NURSINGKING.COM NURSINGKING.COM Determining the woman's level of knowledge concerning contraception and her commitment to any particular method is the primary step of this nursing assessment and necessary before completing the process and moving on to a nursing diagnosis. Once the client's level of knowledge is determined, the nurse can interact with the woman to compare options, reliability, cost, comfort level, protection from STIs, and her partner's willingness to participate. Although important, obtaining data about the frequency of coitus is not the first action that the nurse should undertake when completing an assessment. Data should include not only the frequency of coitus but also the number of sexual partners, level of contraceptive involvement, and the partner's objections. Assessing the woman's willingness to touch herself is a key factor for the nurse to discuss should the client express an interest in using one of the fertility awareness methods of contraception. The nurse must be aware of the client's plan regarding whether she is attempting to prevent conception, delay conception, or conceive.

Which information is most important to provide to the client interested in using the lactational amenorrhea method for contraception? a. LAM is effective until the infant is 9 months of age. b. This popular method of birth control works best if the mother is exclusively breastfeeding. c. Its typical failure rate is 5%. d. Feeding intervals should be 6 hours during the day.

ANS: B The LAM works best if the mother is exclusively or almost exclusively breastfeeding. Disruption of the breastfeeding pattern increases the risk of pregnancy. After the infant is 6 months of age or menstrual flow has resumed, effectiveness decreases. The typical failure rate is 1% to 2%. Feeding intervals should be no greater than 4 hours during the day and 6 hours at night.

A client's oncologist has just finished explaining the diagnostic workup results to her, and she still has questions. The woman states, "The physician says I have a slow-growing cancer. Very few cells are dividing. How does she know this?" What is the name of the test that gave the health care provider this information? a. Tumor ploidy b. S-phase index c. Nuclear grade d. Estrogen-receptor assay

ANS: B The S-phase index measures the number of cells in the synthesis phase of cell development. If the number of cells noted is high, then the cancer is growing at a fast rate. In this client's case, her S-phase index is assumed to be low. Tumor ploidy is the amount of deoxyribonucleic acid (DNA) in a tumor cell, compared with that in a normal cell. Nuclear grade describes the degree of abnormalities present in the cancer cell tubules, the nuclei morphologic features, and mitotic rates. Estrogen and progesterone receptors are proteins found in the cell cytoplasm and surface of some breast cancer cells

During the physical examination of a client beginning prenatal care, which initial action is most important for the nurse to perform? a. Only women who show physical signs or meet the sociologic profile should be assessed for physical abuse. b. The client should empty her bladder before the pelvic examination. c. The distribution, amount, and quality of body hair are of no particular importance. d. The size of the uterus is discounted in the initial examination because it will be increasing in size during the second trimester.

ANS: B The nurse should instruct the client to empty her bladder. An empty bladder facilitates the examination and also provides an opportunity to obtain a urine sample for a number of tests. All women should be assessed for a history of physical abuse, particularly because the likelihood of abuse increases during pregnancy. Noting body hair is important because body hair reflects nutritional status, endocrine function, and hygiene. Particular attention is paid to the size of the uterus because it is an indication of the duration of gestation.

Cardiac output increases up to 50% by the 32nd week of pregnancy. What is the rationale for this change? a. To compensate for the decreased renal plasma flow b. To provide adequate perfusion of the placenta c. To eliminate metabolic wastes of the mother d. To prevent maternal and fetal dehydration

ANS: B The primary function of increased vascular volume is to transport oxygen and nutrients to the fetus via the placenta. Renal plasma flow increases during pregnancy. Assisting with pulling metabolic wastes from the fetus for maternal excretion is one purpose of the increased vascular volume

Of which physiologic alteration of the uterus during pregnancy is it important for the nurse to alert the patient? a. Lightening occurs near the end of the second trimester as the uterus rises into a different position. b. Woman's increased urinary frequency in the first trimester is the result of exaggerated uterine anti-reflexion caused by softening. c. Braxton Hicks contractions become more painful in the third trimester, particularly if the woman tries to exercise. d. Uterine souffle is the movement of the fetus.

ANS: B The softening of the lower uterine segment is called the Hegar sign. In this position, the uterine fundus presses on the bladder, causing urinary frequency that is a normal change of pregnancy. Lightening occurs in the last 2 weeks of pregnancy, when the fetus descends. Braxton Hicks contractions become more defined in the final trimester but are not painful. Walking or exercise usually causes them to stop. The uterine souffle is the sound made by blood in the uterine arteries; it can be heard with a fetal stethoscope.

Importantly, the nurse must be aware of which information related to the use of intrauterine devices (IUDs)? a. Return to fertility can take several weeks after the device is removed. b. Copper IUDs can serve as an emergency contraception under certain situations. c. IUDs offer the same protection against STIs as the diaphragm. d. Consent forms are not needed for IUD insertion.

ANS: B The woman has up to 5 days to insert the IUD after unprotected sex. The return to fertility is immediate after the removal of the IUD. IUDs offer no protection against STIs. A consent form is required for insertion, as is a negative pregnancy test.

The nurse is providing health education to a pregnant client regarding the cardiovascular system. Which information is correct and important to share? a. A pregnant woman experiencing disturbed cardiac rhythm requires close medical and obstetric observation no matter how healthy she may appear otherwise. b. Changes in heart size and position and increases in blood volume create auditory changes from 20 weeks of gestation to term. c. Palpitations are twice as likely to occur in twin gestations. d. All of the above changes will likely occur.

ANS: B These auscultatory changes should be discernible after 20 weeks of gestation. A healthy woman with no underlying heart disease does not need any therapy. The maternal heart rate increases in the third trimester, but palpitations may not necessarily occur, let alone double. Auditory changes are discernible at 20 weeks of gestation.

Which statement regarding multifetal pregnancy is incorrect? a. The expectant mother often develops anemia because the fetuses have a greater demand for iron. b. Twin pregnancies come to term with the same frequency as single pregnancies. c. The mother should be counseled to increase her nutritional intake and gain more weight. d. Backache and varicose veins often are more pronounced with a multifetal pregnancy.

ANS: B Twin pregnancies often end in prematurity. Serious efforts should be made to bring the pregnancy to term. A woman with a multifetal pregnancy often develops anemia, suffers more or worse backache, and needs to gain more weight. Counseling is needed to help her adjust to these conditions.

Which test is performed around the time of ovulation to diagnose the basis of infertility? a. Hysterosalpingogram b. Vaginal ultrasonography c. Laparoscopy d. Follicle-stimulating hormone (FSH) level

ANS: B Ultrasonography is performed around the time of ovulation to assess pelvic structures for abnormalities, to verify follicular development, and to assess the thickness of the endometrium. A hysterosalpingogram is scheduled 2 to 5 days after menstruation to avoid flushing a potentially fertilized ovum out through a uterine tube into the peritoneal cavity. Laparoscopy is usually scheduled early in the menstrual cycle. Hormone analysis is performed to assess endocrine function of the hypothalamic-pituitary-ovarian axis when menstrual cycles are absent or irregular.

What information is important for the nurse to include in planning for the care of a woman who has had a vaginal hysterectomy? a. Expect to be fully recovered in 4 to 6 weeks. b. Expect no changes in hormone levels. c. Expect surgical menopause. d. Take tub baths to aid in healing.

ANS: B Unless the ovaries were also removed, hormonal levels should not change. Menses will cease, but the hypothalamus-pituitary-ovarian axis remains intact. The woman should expect to have vaginal discharge for 4 to 6 weeks. Full recovery varies from woman to woman, depending on risk factors and individual healing. Surgical menopause occurs only if the ovaries are also removed. The client should avoid tub baths, intercourse, and douching until after the follow-up examination

A client in late middle age who is certain she is not pregnant tells the nurse during an office visit that she has urinary problems, as well as sensations of bearing down and of something in her vagina. What condition would the nurse suspect based upon this report? a. Pelvic relaxation b. Cystocele c. Uterine prolapse d. Genital fistulas

ANS: B Uterine displacement can be caused by congenital or acquired weakness of the pelvic support structures and is known as pelvic relaxation. Cystoceles are protrusions of the bladder downward into the vagina; rectoceles are herniations of the anterior rectal wall through a relaxed or ruptured vaginal fascia. Both can produce a bearing-down sensation with urinary dysfunction. They occur more often in older women who have borne children. Uterine prolapse is a more serious type of displacement. In women with a complete prolapse, the cervix and body of the uterus protrude through the vagina. Genital fistulas are perforations between genital tract organs. Most occur between the bladder and the genital tract.

Women who have undergone an oophorectomy, have ovarian failure, or a genetic defect may be eligible to receive donor oocytes (eggs). Which statements regarding oocyte donation are accurate? (Select all that apply.) a. Donor is inseminated with semen from the parent. b. Donor eggs can be fertilized with the male partner's sperm. c. Donors are under 35 years of age. d. Recipient undergoes hormonal stimulation. e. Ovum is placed into a surrogate.

ANS: B, C, D Oocyte donation is usually provided by healthy women under the age of 35 years, who are recruited and paid to undergo ovarian stimulation and oocyte retrieval. The donor eggs are fertilized in a laboratory with the male partner's sperm. The woman undergoes hormonal stimulation to allow the development of the uterine lining. Embryos are then transferred. A donor that is inseminated with the male partner's semen or receives the fertilized ovum and then carries it to gestation is known as a surrogate mother.

Leiomyomas are benign tumors arising from the muscle tissue of the uterus. Which information related to these tumors is accurate? (Select all that apply.) a. Are rapid growing b. Are more common in African-American women c. Are more common in women who have never been pregnant d. Obesity is a risk factor with leiomyomas e. Become malignant if left untreated

ANS: B, C, D The exact cause of leiomyomas remains unknown, although genetic factors may be involved in their development. Most are found in the body of uterus and are classified according to their location on the uterine wall. They are benign, slow growing, and often spontaneously shrink after menopause

Which statement regarding the development of the respiratory system is a high priority for the nurse to understand? a. The respiratory system does not begin developing until after the embryonic stage. b. The infant's lungs are considered mature when the L/S ratio is 1:1, at approximately 32 weeks of gestation. c. Maternal hypertension can reduce maternal-placental blood flow, accelerating lung maturity. d. Fetal respiratory movements are not visible on ultrasound scans until at least 16 weeks of gestation.

ANS: C A reduction in placental blood flow stresses the fetus, increases blood levels of corticosteroids, and thus accelerates lung maturity. The development of the respiratory system begins during the embryonic phase and continues into childhood. The infant's lungs are considered mature when the L/S ratio is 2:1, at approximately 35 weeks of gestation. Lung movements have been visualized on ultrasound scans at 11 weeks of gestation.

Which client would be an ideal candidate for injectable progestins such as medroxyprogesterone acetate as a contraceptive choice? a. The ideal candidate wants menstrual regularity and predictability. b. The client has a history of thrombotic problems or breast cancer. c. The ideal candidate has difficulty remembering to take oral contraceptives daily. d. The client is homeless or mobile and rarely receives health care.

ANS: C Advantages of medroxyprogesterone acetate includes its contraceptive effectiveness, compared with the effectiveness of combined oral contraceptives, and the requirement of only four injections a year. The disadvantages of injectable progestins are prolonged amenorrhea and uterine bleeding. The use of injectable progestin carries an increased risk of venous thrombosis and thromboembolism. To be effective, injections must be administered every 11 to 13 weeks. Access to health care is necessary to prevent pregnancy or potential complications

Which client is most at risk for fibroadenoma of the breast? a. 38-year-old woman b. 50-year-old woman c. 16-year-old girl d. 27-year-old woman

ANS: C Although it may occur at any age, fibroadenoma is most common in the teenage years. Ductal ectasia and intraductal papilloma become more common as a woman approaches menopause. Fibrocystic breast changes are more common during the reproductive years.

Which nursing diagnosis fails to address the psychologic effect of these disorders of a client diagnosed with a neoplasm of the reproductive system? a. Anxiety related to surgical procedures b. Reduced self-concept, resulting from changes in anatomy c. Potential for injury, related to lack of skill for self-care d. Inadequate family processes

ANS: C Although potential for injury, related to lack of skill for self-care, is appropriate to this client's condition, this diagnosis is more suited to the client's learning needs than the psychologic effect. Anxiety, related to surgical procedures, is appropriate for addressing psychosocial concerns; the client may also develop anxiety related to the diagnosis and prognosis and whether or not surgery is required. Reduced self-concept is an applicable diagnosis; changes in her anatomy and function may also result in low self-esteem and ineffective coping skills. Inadequate family processes is a possible and acceptable diagnosis; functional and anatomic changes may result in the client's inability to fulfill her familial role. Depending on the severity of her condition, inadequate family processes could also lead to social isolation

Some pregnant clients may report changes in their voice and impaired hearing. What should the nurse explain to the client concerning these findings? a. Voice changes are caused by decreased estrogen levels. b. Displacement of the diaphragm results in thoracic breathing. c. These changes are the results of congestion and swelling of the upper respiratory tract. d. Increased blood volume causes changes in the voice.

ANS: C Although the diaphragm is displaced, and the volume of blood is increased, neither causes changes in the voice nor impairs hearing. The key is that estrogen levels increase, not decrease, which causes the upper respiratory tract to become more vascular, which produces swelling and congestion in the nose and ears and therefore voice changes and impaired hearing.

During the initial visit with a client who is beginning prenatal care, which action should be the highest priority for the nurse? a. The first interview is a relaxed, get-acquainted affair during which the nurse gathers some general impressions of his or her new client. b. If the nurse observed handicapping conditions, he or she should be sensitive and not inquire about them because the client will do that in her own time. c. The nurse should be alert to the appearance of potential parenting problems, such as depression or lack of family support. d. Because of legal complications, the nurse should not ask about illegal drug use; that is left to the physician.

ANS: C Besides these potential problems, the nurse needs to be alert to the woman's attitude toward keeping regular health care appointments. If the client lacks insurance, then the nurse may be able to direct her to resources that provide assistance for pregnant women (i.e., Women, Infants, and Children [WIC]; Medicaid). The initial interview needs to be planned, purposeful, and focused on specific content. A lot of ground must be covered. The nurse must be sensitive to special problems; he or she should inquire because discovering individual needs is important. A client with a chronic or handicapping condition might forget to mention it because she has adapted to it. Obtaining information on drug use is important and can be confidentially done. Actual testing for drug use requires the client's consent.

Breast pain occurs in many women during their perimenopausal years. Which information is a priority for the nurse to share with the client? a. Breast pain is an early indication of cancer. b. Pain is almost always an indication of a solid mass. c. Distinguishing between cyclical and noncyclical pain is important. d. Breast pain is most often treated with narcotics.

ANS: C Breast pain is unusual in breast cancer. Solid masses are generally benign and described as smooth, round, mobile, and painless. Distinguishing between cyclical and noncyclical pain is important to determine whether the cause is hormonal. Idiopathic pain is most often treated with nonsteroidal anti-inflammatory medications.

A nurse is providing breast care education to a client after mammography. Which information regarding fibrocystic changes in the breast is important for the nurse to share? a. Fibrocystic breast disease is a disease of the milk ducts and glands in the breasts. b. It is a premalignant disorder characterized by lumps found in the breast tissue. c. Fibrocystic breast disease causes lumpiness with or without pain and tenderness in varying degrees in the breast tissue during menstrual cycles. d. Lumpiness is accompanied by tenderness after menses.

ANS: C Fibrocystic changes are palpable thickenings in the breast and can be associated with or without pain and tenderness. The symptoms fluctuate with the menstrual cycle. Fibrocystic changes are not premalignant changes; this information is inaccurate. Tenderness most often occurs before menses.

A myomectomy is a procedure prescribed for symptom relief of what condition? a. Numerous small fibroid tumors b. Bartholin cysts c. Fibroid tumors near the outer wall of the uterus with a uterine size no larger than at 12 weeks of gestation d. Leiomyomas in a uterus larger than 14 weeks of gestation

ANS: C If a fibroid tumor lies near the outer wall of the uterus and the uterine size is no larger than at 12 to 14 weeks of gestation and the symptoms are significant, a myomectomy (i.e., removal of the tumor) may be performed. This procedure leaves the uterine walls relatively intact, thereby preserving the uterus for future pregnancies. Laser surgery or electrocauterization can be safely used to destroy numerous small fibroid tumors. Bartholin cysts are benign lesions of the vulva. If the cyst is symptomatic or infected, surgical incision and drainage may provide relief. A hysterectomy (i.e., removal of the entire uterus) is the treatment of choice if bleeding is severe or if the fibroid tumor is obstructing the normal function of other organs.

In the acronym BRAIDED, which letter is used to identify the key components of informed consent that the nurse must document? a. B stands for birth control. b. R stands for reproduction. c. A stands for alternatives. d. I stands for ineffective.

ANS: C In the acronym BRAIDED, A stands for alternatives and information about other viable methods. B stands for benefits and information about the advantages of a particular birth control method and its success rates. R stands for risks and information about the disadvantages of a particular method and its failure rates. I stands for inquiries and the opportunity to ask questions.

A woman has a breast mass that is not well delineated and is nonpalpable, immobile, and nontender. Which condition is this client experiencing? a. Fibroadenoma b. Lipoma c. Intraductal papilloma d. Mammary duct ectasia

ANS: C Intraductal papilloma is the only benign breast mass that is nonpalpable. Fibroadenoma is well delineated, palpable, and movable. Lipoma is palpable and movable. Mammary duct ectasia is not well delineated and is immobile, but it is palpable and painful.

The nurse is providing education to a client regarding the normal changes of the breasts during pregnancy. Which statement regarding these changes is correct? a. The visibility of blood vessels that form an intertwining blue network indicates a possible infection of the tubercles. b. The mammary glands do not develop until 2 weeks before labor. c. Lactation is inhibited until the progesterone level declines after birth. d. Colostrum is the yellowish oily substance used to lubricate the nipples for breastfeeding

ANS: C Lactation is inhibited until after birth due to progesterone levels. The visible blue network of blood vessels is a normal outgrowth of a richer blood supply. The mammary glands are functionally complete by mid-pregnancy. Colostrum is a creamy white-to-yellow pre-milk fluid that can be expressed from the nipples before birth.

What should the nurse be cognizant of concerning the client's reordering of personal relationships during pregnancy? a. Because of the special motherhood bond, a woman's relationship with her mother is even more important than with her partner. b. Nurses need not get involved in any sexual issues the couple has during pregnancy, particularly if they have trouble communicating them to each other. c. Women usually express two major relationship needs during pregnancy: feeling loved and valued and having the child accepted by the partner. d. The woman's sexual desire is likely to be highest in the first trimester because of the excitement and because intercourse is physically easier.

ANS: C Love and support help a woman feel better about her pregnancy. The most important person to the pregnant woman is usually her partner. Nurses can facilitate communication between partners about sexual matters if, as is common, they are nervous about expressing their worries and feelings to one another. The second trimester is the time when a woman's sense of well-being, along with certain physical changes, increases her desire for sex. Sexual desire is down in the first and third trimesters.

The nurse is providing contraceptive instruction to a young couple who are eager to learn. The nurse should be cognizant of which information regarding the natural family planning method? a. The natural family planning method is the same as coitus interruptus or "pulling out." b. This contraception method uses the calendar method to align the woman's cycle with the natural phases of the moon. c. This practice is the only contraceptive method acceptable to the Roman Catholic Church. d. The natural family planning method relies on barrier methods during the fertility phases.

ANS: C Natural family planning is the only contraceptive practice acceptable to the Roman Catholic Church. "Pulling out" is not the same as periodic abstinence, another name for natural family planning. The phases of the moon are not part of the calendar method or any method. Natural family planning is another name for periodic abstinence, which is the accepted way to pass safely through the fertility phases without relying on chemical or physical barriers.

Which condition does NOT increase a client's risk for breast cancer? a. BRCA1 or BRCA2 gene mutation b. Li-Fraumeni syndrome c. Paget disease d. Cowden syndrome

ANS: C Paget disease originates in the nipple and causes nipple carcinoma and exhibits bleeding, oozing, and crusting of the nipple. BRCA1 or BRCA2, Li-Fraumeni syndrome, and Cowden syndrome are all genetic mutations that have different family pedigrees and increase the risk of breast cancer.

Which clinical finding in a primiparous client at 32 weeks of gestation might be an indication of anemia? a. Ptyalism b. Pyrosis c. Pica d. Decreased peristalsis

ANS: C Pica (a desire to eat nonfood substances) is an indication of iron deficiency and should be evaluated. Cravings include ice, clay, and laundry starch. Ptyalism (excessive salivation), pyrosis (heartburn), and decreased peristalsis are normal findings.

The measurement of lecithin in relation to sphingomyelin (lecithin/sphingomyelin [L/S] ratio) is used to determine fetal lung maturity. Which ratio reflects fetal maturity of the lungs? a. 1.4:1 b. 1.8:1 c. 2:1 d. 1:1

ANS: C The L/S ratio indicates a 2:1 ratio of lecithin to sphingomyelin, which is an indicator of fetal lung maturity and occurs at approximately the middle of the third trimester. L/S ratios of 1.4:1, 1.8:1, and 1:1 each indicate immaturity of the fetal lungs.

A woman's cousin gave birth to an infant with a congenital heart anomaly. The woman asks the nurse when such anomalies occur during development. Which response by the nurse is most accurate? a. "We don't really know when such defects occur." b. "It depends on what caused the defect." c. "Defects occur between the third and fifth weeks of development." d. "They usually occur in the first 2 weeks of development."

ANS: C The cardiovascular system is the first organ system to function in the developing human. Blood vessel and blood formation begins in the third week, and the heart is developmentally complete in the fifth week. "We don't really know when such defects occur" is an inaccurate statement. Regardless of the cause, the heart is vulnerable during its period of development—in the third to fifth weeks; therefore, the statement, "They usually occur in the first 2 weeks of development" is inaccurate.

A client arrives for her initial prenatal examination. This is her first child. She asks the nurse, "How does my baby get air inside my uterus?" What is the correct response by the nurse? a. "The baby's lungs work in utero to exchange oxygen and carbon dioxide." b. "The baby absorbs oxygen from your blood system." c. "The placenta provides oxygen to the baby and excretes carbon dioxide into your bloodstream." d. "The placenta delivers oxygen-rich blood through the umbilical artery to the baby's abdomen."

ANS: C The placenta delivers oxygen-rich blood through the umbilical vein, not the artery, to the fetus and excretes carbon dioxide into the maternal bloodstream. The fetal lungs do not function as respiratory gas exchange in utero. The baby does not simply absorb oxygen from a woman's blood system; rather, blood and gas transport occur through the placenta.

Dental care during pregnancy is an important component of good prenatal care. Which instruction regarding dental health should the nurse provide? a. Regular brushing and flossing may not be necessary during early pregnancy because it may stimulate the woman who is already nauseated to vomit. A cleaning is all that is necessary. b. Dental surgery is contraindicated during pregnancy and should be delayed until after delivery. c. If dental treatment is necessary, then the woman will be most comfortable with it in the second trimester. d. If a woman has dental anxiety, then dental care may interfere with the expectant mother's need to practice conscious relaxation and to prepare for labor.

ANS: C The second trimester is the best time for dental treatment because the woman will be able to sit most comfortably in the dental chair. Dental care, such as brushing with a fluoride toothpaste, is especially important during pregnancy. Periodontal disease has been linked to both preterm labor and low-birth-weight (LBW) infants. Emergency dental surgery is permissible; however, the mother must clearly understand the risks and benefits. Conscious relaxation is useful and may even help the woman get through any dental appointments, but it is not a reason to avoid them.

Which consideration is essential for the nurse to understand regarding follow-up prenatal care visits? a. The interview portions become more intensive as the visits become more frequent over the course of the pregnancy. b. Monthly visits are scheduled for the first trimester, every 2 weeks for the second trimester, and weekly for the third trimester. c. During the abdominal examination, the nurse should be alert for supine hypotension. d. For pregnant women, a systolic BP of 130 mm Hg and a diastolic BP of 80 mm Hg is sufficient to be considered hypertensive.

ANS: C The woman lies on her back during the abdominal examination, possibly compressing the vena cava and aorta, which can cause a decrease in BP and a feeling of faintness. The interview portion of the follow-up examinations is less extensive than in the initial prenatal visits, during which so much new information must be gathered. Monthly visits are routinely scheduled for the first and second trimesters; visits increase to every 2 weeks at week 28 and to once a week at week 36. For pregnant women, hypertension is defined as a systolic BP of 140 mm Hg or higher and a diastolic BP of 90 mm Hg or higher.

A couple comes in for an infertility workup, having attempted to achieve pregnancy for 2 years. The woman, 37 years of age, has always had irregular menstrual cycles but is otherwise healthy. The man has fathered two children from a previous marriage and had a vasectomy reversal 2 years ago. The man has had two normal semen analyses, but the sperm seem to be clumped together. What additional testing is needed? TEST BANK FOR MATERNITY AND WOMEN'S HEALTH CARE 12TH EDITION BY LOWDERMILK NURSINGKING.COM NURSINGKING.COM a. Testicular biopsy b. Antisperm antibodies c. Serum prolactin level d. Examination for testicular infection

ANS: C This scenario does not indicate that the woman has had any testing related to her irregular menstrual cycles. Hormone analysis is performed to assess endocrine function of the hypothalamic-pituitary-ovarian axis when menstrual cycles are absent or irregular. Determining the blood levels of prolactin, FSH, luteinizing hormone (LH), estradiol, progesterone, and thyroid hormones may be necessary to diagnose the cause of the woman's irregular menstrual cycles. A testicular biopsy is indicated only in cases of azoospermia (no sperm cells) or severe oligospermia (low number of sperm cells). Although unlikely to be the case because the husband has already produced children, antisperm antibodies may be produced by the man against his own sperm. Examination for testicular infection would be performed before semen analysis. Furthermore, infection would affect spermatogenesis.

A woman will be taking oral contraceptives using a 28-day pack. What advice should the nurse provide to protect this client from an unintended pregnancy? a. Limit sexual contact for one cycle after starting the pill. b. Use condoms and foam instead of the pill for as long as the client takes an antibiotic. c. Take one pill at the same time every day. d. Throw away the pack and use a backup method if two pills are missed during week 1 of her cycle.

ANS: C To maintain adequate hormone levels for contraception and to enhance compliance, clients should take oral contraceptives at the same time each day. If contraceptives are to be started at any time other than during normal menses or within 3 weeks after birth or an abortion, then another method of contraception should be used through the first week to prevent the risk of pregnancy. Taken exactly as directed, oral contraceptives prevent ovulation, and pregnancy cannot occur. No strong pharmacokinetic evidence indicates a link between the use of broad-spectrum antibiotics and altered hormonal levels in oral contraceptive users. If the client misses two pills during week 1, then she should take two pills a day for 2 days and finish the package and use a backup contraceptive method for the next 7 consecutive days.

A pregnant woman at 10 weeks of gestation jogs three or four times per week. She is concerned about the effect of the exercise on the fetus. Which guidance should the nurse provide? a. "You don't need to modify your exercising any time during your pregnancy." b. "Stop exercising because it will harm the fetus." c. "You may find that you need to modify your exercise to walking later in your pregnancy, around the seventh month." d. "Jogging is too hard on your joints; switch to walking now."

ANS: C Typically, running should be replaced with walking around the seventh month of pregnancy. The nurse should inform the woman that she may need to reduce her exercise level as the pregnancy progresses. Physical activity promotes a feeling of well-being in pregnant women. It improves circulation, promotes relaxation and rest, and counteracts boredom. Simple measures should be initiated to prevent injuries, such as warm-up and stretching exercises to prepare the joints for more strenuous exercise.

A woman arrives at the clinic for a pregnancy test. Her last menstrual period (LMP) was February 14, 2019. What is the client's expected date of birth (EDB)? a. September 17, 2019 b. November 7, 2019 c. November 21, 2019 d. December 17, 2019

ANS: C Using the Nägele's rule, the EDB is calculated by subtracting 3 months from the month of the LMP and adding 7 days + 1 year to the day of the LMP. Therefore, with an LMP of February 14, 2019, her due date is November 21, 2019. September 17, 2019 is too short a period to complete a normal pregnancy. Using the Nägele's rule, an EDB of November 7, 2019, is 2 weeks early. December 17, 2019 is almost a month past the correct EDB.

A couple is attempting to cope with an infertility problem. They want to know what they can do to preserve their emotional equilibrium. What is the nurse's most appropriate response? a. "Tell your friends and family so that they can help you." b. "Talk only to other friends who are infertile, because only they can help." c. "Get involved with a support group. I'll give you some names." d. "Start adoption proceedings immediately, because adopting an infant can be very difficult."

ANS: C Venting negative feelings may unburden the couple. A support group may provide a safe haven for the couple to share their experiences and gain insight from others' experiences. Although talking about their feelings may unburden them of negative feelings, infertility can be a major stressor that affects the couple's relationships with family and friends. Limiting their interactions to other infertile couples may be a beginning point for addressing psychosocial needs. However, depending on where the other couple is in their own recovery process, limiting their interactions may not be of assistance to them. Telling the couple to start adoption proceedings immediately is not supportive of the psychosocial needs of this couple and may be detrimental to their well-being.

Which procedure falls into the category of micromanipulation techniques of the follicle? (Select all that apply.) a. Intrauterine insemination b. Preimplantation genetic diagnosis c. Intracytoplasmic sperm injection (ISCI) d. Assisted hatching e. IVF-ET

ANS: C, D ISCI makes it possible to achieve fertilization even with a few or poor-quality sperm by introducing sperm beneath the zone pellucid into the egg. Another micromanipulation technique is assisted hatching. An infrared laser breaks through the thick or tough zone pellucid, enabling the blastocyst to hatch.

During pregnancy, many changes occur as a direct result of the presence of the fetus. Which of these adaptations meet this criterion? (Select all that apply.) a. Leukorrhea b. Development of the operculum c. Quickening d. Ballottement e. Lightening

ANS: C, D, E Quickening is the first recognition of fetal movements or "feeling life." Quickening is often described as a flutter and is felt earlier in the multiparous woman than in the primiparous woman. Passive movement of the unengaged fetus is referred to as ballottement. Lightening occurs when the fetus begins to descend into the pelvis and occurs 2 weeks before labor in the nulliparous woman and at the start of labor in the multiparous woman. Leukorrhea is a white or slightly gray vaginal discharge that develops in response to cervical stimulation by estrogen and progesterone. Mucus fills the cervical canal creating a plug otherwise known as the operculum. The operculum acts as a barrier against bacterial invasion during the pregnancy.

A pregnant couple has formulated a birth plan and is reviewing it with the nurse at an expectant parent's class. Which aspect of their birth plan should be considered potentially unrealistic and require further discussion with the nurse? a. "My partner and I have agreed that my sister will be my coach because he becomes anxious with regard to medical procedures and blood. He will be nearby and check on me every so often to make sure everything is okay." b. "We plan to use the techniques taught in the Lamaze classes to reduce the pain experienced during labor." c. "We want the labor and birth to take place in a birthing room. My partner will come in the minute the baby is born." d. "Regardless of the circumstances, we do not want the fetal monitor used during labor because it will interfere with movement and doing effleurage."

ANS: D Because monitoring is essential to assess fetal well-being, fetal monitoring is not a factor that can be determined by the couple. The nurse should fully explain its importance. The option for intermittent electronic monitoring could be explored if this is a low-risk pregnancy and as long as labor is normally progressing. The birth plan is a tool with which parents can explore their childbirth options; however, the plan must be viewed as tentative. Having the woman's sister as her coach with her husband nearby is an acceptable request for a laboring woman. Using breathing techniques to alleviate pain is a realistic part of a birth plan. Not all fathers are able to be present during the birth; however, this couple has made a realistic plan that works for their specific situation.

Which structure is responsible for oxygen and carbon dioxide transport to and from the maternal bloodstream? a. Decidua basalis b. Blastocyst c. Germ layer d. Chorionic villi

ANS: D Chorionic villi are fingerlike projections that develop out of the trophoblast and extend into the blood-filled spaces of the endometrium. The villi obtain oxygen and nutrients from the maternal bloodstream and dispose carbon dioxide and waste products into the maternal blood. The decidua basalis is the portion of the decidua (endometrium) under the blastocyst where the villi attach. The blastocyst is the embryonic development stage after the morula; implantation occurs at this stage. The germ layer is a layer of the blastocyst.

A man smokes two packs of cigarettes a day. He wants to know if smoking is contributing to the difficulty he and his wife are having getting pregnant. Which guidance should the nurse provide? a. "Your sperm count seems to be okay in the first semen analysis." b. "Only marijuana cigarettes affect sperm count." c. "Although smoking has no effect on sperm count, it can give you lung cancer." d. "Smoking can reduce the quality of your sperm."

ANS: D Cigarette smoking has detrimental effects on sperm and has been associated with abnormal sperm, a decreased number of sperm, and chromosomal damage. The nurse may suggest a smoking cessation program to increase the fertility of the male partner. Sperm counts vary from day to day and are dependent on emotional and physical status and sexual activity. Therefore, a single analysis may be inconclusive. A minimum of two analyses must be performed several weeks apart to assess male fertility. Marijuana use may depress the number and motility of sperm. Smoking is indeed a causative agent for lung cancer.

To reassure and educate their pregnant clients regarding changes in their blood pressure, nurses should be cognizant of what? a. A blood pressure cuff that is too small produces a reading that is too low; a cuff that is too large produces a reading that is too high. b. Shifting the client's position and changing from arm to arm for different measurements produces the most accurate composite blood pressure reading at each visit. c. Systolic blood pressure slightly increases as the pregnancy advances; diastolic pressure remains constant. d. Compression of the iliac veins and inferior vena cava by the uterus contributes to hemorrhoids in the later stage of a term pregnancy.

ANS: D Compression of the iliac veins and inferior vena cava by the uterus contributes to hemorrhoids in the later stage of a term pregnancy. This compression also leads to varicose veins in the legs and vulva. The tightness of a blood pressure cuff that is too small produces a reading that is too high; similarly, the looseness of a cuff that is too large results in a reading that is too low. Because maternal positioning affects readings, blood pressure measurements should be obtained in the same arm and with the woman in the same position. The systolic blood pressure generally remains constant but may decline slightly as the pregnancy advances. The diastolic blood pressure first decreases and then gradually increases.

Which statement regarding the term contraceptive failure rate is the most accurate? a. The contraceptive failure rate refers to the percentage of users expected to have an accidental pregnancy over a 5-year span. b. It refers to the minimum rate that must be achieved to receive a government license. c. The contraceptive failure rate increases over time as couples become more careless. d. It varies from couple to couple, depending on the method and the users.

ANS: D Contraceptive effectiveness varies from couple to couple, depending on how well a contraceptive method is used and how well it suits the couple. The contraceptive failure rate measures the likelihood of accidental pregnancy in the first year only. Failure rates decline over time because users gain experience.

A woman who is 8 months pregnant asks the nurse, "Does my baby have any antibodies to fight infection?" What is the most appropriate response by the nurse? a. "Your baby has all the immunoglobulins necessary: immunoglobulin G (IgG), immunoglobulin M (IgM), and immunoglobulin A (IgA)." b. "Your baby won't receive any antibodies until he is born and you breastfeed him." c. "Your baby does not have any antibodies to fight infection." d. "Your baby has IgG and IgM."

ANS: D During the third trimester, IgG is the only immunoglobulin that crosses the placenta; it provides passive acquired immunity to specific bacterial toxins. However, the fetus produces IgM by the end of the first trimester. IgA immunoglobulins are not produced by the baby. Therefore, by the third trimester, the fetus has both IgG and IgM. Breastfeeding supplies the newborn infant with IgA.

A woman is 3 months pregnant. At her prenatal visit she tells the nurse that she does not know what is happening; one minute she is happy that she is pregnant and the next minute she cries for no reason. Which response by the nurse is most appropriate? a. "Don't worry about it; you'll feel better in a month or so." b. "Have you talked to your husband about how you feel?" c. "Perhaps you really don't want to be pregnant." d. "Hormone changes during pregnancy commonly result in mood swings."

ANS: D Explaining that hormone changes can result in mood swings is an accurate statement and the most appropriate response by the nurse. Telling the woman not to worry dismisses her concerns and is not the most appropriate response. Although the woman should be encouraged to share her feelings, asking if she has spoken to her husband about them is not the most appropriate response and does not provide her with a rationale for the psychosocial dynamics of her pregnancy. Suggesting that the woman does not want to be pregnant is completely inappropriate and deleterious to the psychologic well-being of the woman. Hormonal and metabolic adaptations often cause mood swings in pregnancy. The woman's responses are normal. She should be reassured about her feelings.

hCG is an important biochemical marker for pregnancy and therefore the basis for many tests. Which statement regarding hCG is true? a. hCG can be detected as early as weeks after conception. b. hCG levels gradually and uniformly increase throughout pregnancy. c. Significantly lower-than-normal increases in the levels of hCG may indicate a postdate pregnancy. d. Higher-than-normal levels of hCG may indicate an ectopic pregnancy or Down syndrome.

ANS: D Higher hCG levels also could be a sign of a multiple gestation. hCG can be detected as early as 7 to 10 days after conception. The hCG levels fluctuate during pregnancy, peaking, declining, stabilizing, and then increasing again. Abnormally slow increases may indicate impending miscarriage.

Pregnancy hormones prepare the vagina for stretching during labor and birth. Which change related to the pelvic viscera should the nurse share with the client? a. Because of several changes in the cervix, abnormal Papanicolaou (Pap) tests are easier to evaluate. b. Quickening is a technique of palpating the fetus to engage it in passive movement. c. The deepening color of the vaginal mucosa and cervix (Chadwick sign) usually appears in the second trimester or later as the vagina prepares to stretch during labor. d. Increased vascularity of the vagina increases sensitivity and may lead to a high degree of arousal, especially in the second trimester

ANS: D Increased sensitivity and an increased interest in sex sometimes go together and frequently occur during the second trimester. These cervical changes make evaluation of abnormal Pap tests more difficult. Quickening is the first recognition of fetal movements by the mother. Ballottement is a technique used to palpate the fetus. The Chadwick sign appears from the 6 to 8 weeks of gestation.

Which gastrointestinal alteration of pregnancy is a normal finding? a. Insufficient salivation (ptyalism) is caused by increases in estrogen. b. Acid indigestion (pyrosis) begins early but declines throughout pregnancy. c. Hyperthyroidism often develops (temporarily) because hormone production increases. d. Nausea and vomiting rarely have harmful effects on the fetus and may be beneficial.

ANS: D Normal nausea and vomiting rarely produce harmful effects and may be less likely to result in miscarriage or preterm labor. Ptyalism is excessive salivation that may be caused by a decrease in unconscious swallowing or by stimulation of the salivary glands. Pyrosis begins as early as the first trimester and intensifies through the third trimester. Increased hormone production does not lead to hyperthyroidism in pregnant women.

A healthy 60-year-old African-American woman regularly receives health care at her neighborhood clinic. She is due for a mammogram. At her first visit, her health care provider is concerned about the 3-week wait at the neighborhood clinic and made an appointment for her to have a mammogram at a teaching hospital across town. She did not keep her appointment and returned to the clinic today to have the nurse check her blood pressure. What is the most appropriate statement for the nurse to make to this client? a. "Do you have transportation to the teaching hospital so that you can get your mammogram?" b. "I'm concerned that you missed your appointment; let me make another one for you." c. "It's very dangerous to skip your mammograms; your breasts need to be checked." d. "Would you like me to make an appointment for you to have your mammogram here?"

ANS: D Offering to make an appointment for the client at the neighborhood location is nonjudgmental and gives her options as to where she may have her mammogram. Furthermore, it is an innocuous way to investigate the reasons the client missed her previous appointment. Mortality rates from breast cancer remain high for African-American women. Rather than reminding this woman that she has "missed her appointment," discussing the evidence behind the recommendations for a mammogram might be preferable for the nurse. The nurse can offer to reschedule should the client agree to return for the test. Telling the client that it is dangerous to skip mammograms can be perceived as judgmental and derogatory and may alienate and embarrass the client.

Many pregnant women have questions regarding work and travel during pregnancy. Which education is a priority for the nurse to provide? a. Women should sit for as long as possible and cross their legs at the knees from time to time for exercise. b. Women should avoid seat belts and shoulder restraints in the car because they press on the fetus. c. Metal detectors at airport security checkpoints can harm the fetus if the woman passes through them several times. d. While working or traveling in a car or on an airplane, women should arrange to walk around at least every hour or so.

ANS: D Periodic walking helps prevent thrombophlebitis. Pregnant women should avoid sitting or standing for long periods and crossing the legs at the knees. Pregnant women must wear lap belts and shoulder restraints. The most common injury to the fetus comes from injury to the mother. Metal detectors at airport security checkpoints do not harm fetuses.

Which hormone is essential for maintaining pregnancy? a. Estrogen b. hCG c. Oxytocin d. Progesterone

ANS: D Progesterone is essential for maintaining pregnancy; it does so by relaxing smooth muscles, which reduces uterine activity and prevents miscarriage. Estrogen plays a vital role in pregnancy, but it is not the primary hormone for maintaining pregnancy. hCG levels rise at implantation but decline after 60 to 70 days. Oxytocin stimulates uterine contractions.

Which sign or symptom is considered a first-trimester warning sign and should be immediately reported by the pregnant woman to her health care provider? a. Nausea with occasional vomiting b. Fatigue c. Urinary frequency d. Vaginal bleeding

ANS: D Signs and symptoms that must be reported include severe vomiting, fever and chills, burning on urination, diarrhea, abdominal cramping, and vaginal bleeding. These symptoms may be signs of complications of the pregnancy. Nausea with occasional vomiting is a normal first-trimester complaint. Although it may be worrisome or annoying to the mother, it is not usually an indication of a problem with the pregnancy. Fatigue is common during the first trimester. Because of physiologic changes that happen during pregnancy, clients should be taught that urinary frequency is normal.

Which precaution should the nurse take while caring for a client who is undergoing internal radiation therapy for cervical cancer? a. Wear gloves when assessing the cervical intracavity implant. b. Instruct the client to urinate in the lead-lined bedpan or "hat" every 2 hours. c. Prepare the client for an enema before inserting the implant. d. Limit staff or visitor exposure to 30 minutes or less in an 8-hour period.

ANS: D Staff and visitor exposure should be limited to 30 minutes or less in an 8-hour period to reduce the risk of overexposure to radiation. Nurses need to protect themselves from overexposure to radiation. Wearing a shield is one method of protection. An indwelling catheter is inserted to prevent urinary distention that could dislodge the applicator. No bowel preparation is necessary.

Which woman is at the greatest risk for psychologic complications after hysterectomy? a. 55-year-old woman who has been having abnormal bleeding and pain for 3 years b. 46-year-old woman who has had three children and has just been promoted at work c. 62-year-old widow who has three friends who have had uncomplicated hysterectomies d. 19-year-old woman who had a ruptured uterus after giving birth to her first child

ANS: D The 19-year-old woman is still in her childbearing years. Often the uterus is related to self-concept in women in this age group, and they may feel that sexual functioning is related to having a uterus. The 55-year-old woman is past her childbearing years and has had bleeding and pain for 3 years. The hysterectomy may be well received as a method of pain relief. The 46-year-old woman has a family and positive event occurring in her life (job promotion). The 62-year-old woman is past her reproductive years and has relationships with others who have had positive outcomes.

A woman is using the basal body temperature (BBT) method of contraception. She calls the clinic and tells the nurse, "My period is due in a few days, and my temperature has not gone up." What is the nurse's most appropriate response? a. "This probably means that you're pregnant." b. "Don't worry; it's probably nothing." c. "Have you been sick this month?" d. "You probably didn't ovulate during this cycle."

ANS: D The absence of a temperature decrease most likely is the result of a lack of ovulation. Pregnancy cannot occur without ovulation, which is being measured using the BBT method. A comment such as, "Don't worry; it's probably nothing," discredits the client's concerns. Illness is most likely the cause of an increase in BBT.

A client currently uses a diaphragm and spermicide for contraception. She asks the nurse to explain the major differences between the cervical cap and the diaphragm. What is the most appropriate response by the nurse? a. "No spermicide is used with the cervical cap, so it's less messy." b. "The diaphragm can be left in place longer after intercourse." c. "Repeated intercourse with the diaphragm is more convenient." d. "The cervical cap can be safely used for repeated acts of intercourse without adding more spermicide later."

ANS: D The cervical cap can be inserted hours before sexual intercourse without the need for additional spermicide later. Spermicide should be used inside the cap as an additional chemical barrier. The cervical cap should remain in place for 6 hours after the last act of intercourse. Repeated intercourse with the cervical cap is more convenient because no additional spermicide is needed.

Nurses should be cognizant of what information regarding the non-contraceptive medical effects of combination oral contraceptives (COCs)? a. COCs can cause TSS if the prescription is wrong. b. Hormonal withdrawal bleeding is usually a little more profuse than in normal menstruation and lasts a week for those who use COCs. c. COCs increase the risk of endometrial and ovarian cancers. d. Effectiveness of COCs can be altered by some over-the-counter medications and herbal supplements.

ANS: D The effectiveness of COCs can be altered by some over-the-counter medications and herbal supplements. TSS can occur in some who use the diaphragm, but it is not a consequence of taking oral contraceptive pills. Hormonal withdrawal bleeding usually is lighter than in normal menstruation and lasts a couple of days. Oral contraceptive pills offer protection against the risk of endometrial and ovarian cancers.

Hyperemesis gravidarum

Severe vomiting in early pregnancy


Kaugnay na mga set ng pag-aaral

Year 9 Spanish 40 Words List Module 1

View Set

Stroke, Intracranial Pressure, Increased Intracranial Pressure, HIV, Conflict Resolution

View Set

Med Surge Multiple Sclerosis & Spinal Cord injury

View Set

HESI - practice notes: health and physical assessment

View Set